5.0 Project Planning & Design (COPY)

Lakukan tugas rumah & ujian kamu dengan baik sekarang menggunakan Quizwiz!

What is A-2 Occupancy Group?

Assembly for food and drink consumption

What is A-5 Occupancy Group?

Assembly for outdoor sports

What is A-4 Occupancy Group?

Assembly for viewing of indoor sports

How far can trim or nonstructural horizontal projections project into a corridor that is apart of a means of egress?

1.5"

What is the declination angle of the earth?

23.4°

What is the maximum square footage an opening can have in a fire barrier wall?

25% of the length of the wall and cannot exceed 156 sq ft unless the opening has a special fire-rating

What is the term for lengthwise wood grain separation caused by seasoning? A. Check B. Pitch Pocket C. Shake D. Split

A Lengthwise wood grain separation caused by seasoning is termed a check. A shake is lengthwise grain separation inherent in the wood before seasoning.

What is the term for lengthwise wood grain separation caused by seasoning? A. Check B. Pitch pocket C. Shake D. Split

A Lengthwise wood grain separation caused by seasoning is termed a check. A shake is lengthwise grain separation inherent in the wood before seasoning.

What is R-1 Occupancy Group?

Residential for transient lodging hotels and motels

What are the three weathering grades of brick?

Severe weathering (SW) Moderate weathering (MW) Negligible Weathering (NW)

What are Super Windows?

Glazing units that combine two low-e coatings with gas-filled cavities between three layers of glass. With a U-value of 0.15 Btu/ft²-hr-°F or less, these units can gain more thermal energy than they lose over a 24-hour period in winter.

What is hue, value, and intensity?

Hue is basically the pigment of a color (what distinguishes a color from another). Value is degrees of lightness and darkness. Intensity is defined by the degree of purity of the hue when compared with a neutral gray of the same value.

What is the Munsell Color System?

It defines color more precisely than the Brewster Color System but designating colors with values of lightness to darkness in a 3 dimensional graph.

What are the four types of mortar?

M - 2500 psi S - 1800 psi N - 750 psi O - Interior Walls

What is I-2 Occupancy Group?

Medical care on 24-hour basis

What is M Occupancy Group?

Mercantile

In terms of the coefficient of sound absorption, what ratio is considered reflective and what is considered absorptive?

Reflective - anything below 0.2 Absorptive - anything above 0.2

In general, transmission loss through a barrier tends to increase with the frequency of sound. True of False?

True

What is sheet flow?

Water that drains across a sloping surface.

What are Fluorescent Lamps?

contain a mixture of an inert gas and low-pressure mercury vapor. When a fluorescent lamp is energized, a mercury arc is formed that creates ultraviolet light. This invisible light, in turn, strikes the phosphor-coated bulb, causing the bulb to fluoresce and produce visible light.

What is generally the accepted slope for under ground drainage?

0.3%

What is R-3 Occupancy Group?

1 or 2 dwelling units with attached uses or child care < 6, less than 24-hour care

What are the six categories of whole-building LCA ?

1) Acidificaiton - is the generation of materials that can lower the pH of the surrounding waterways and soil. 2) Eutrophication - is the formation of excessive nutrients in a body of water that promotes algae growth which blocks sunlight below the surface which is bad for water life. 3) Global Warming 4) Ozone Layer Depletion 5) Formation of Ground-Level Ozone 6) Depletion of Non-Renewable Energy Resources

What are the four tests needed to pass in order for a manufacturer to obtain the CRI (Carpet and Rug Institute) IAQ carpet testing program label or "Green Label"?

1) Total volatile organic compounds 2) Styrene 3) Formaldehyde 4) 4-phenylcyclohexene

What is generally the accepted slope for above ground drainage?

1.5%

What is the required exit stair separation according to IBC?

1/2 the distance measured diagonally across the floor plate. The distance can be shortened to 1/3 if sprinklered.

A wall with 1% open area will have a maximum transmission loss of about ________.

20 dB

The word "lamella" refers to a type of __________. A. Aggregate used in concrete B. Arch C. Beam Shape D. Roof Structure

A lamella is a type of roof structure. It's geometry allows it to span large distances with little or no support. Lamella originates from the latin word lamina which means thin plate.

What is a cool roof?

A reflective light-colored roof that has a minimum reflectivity of 0.65 on the first year and a 0.50 reflectivity after three years.

What is a Vierendeel Truss?

A structure composed of a series of rigid rectangular frames.

What is Variable-Voltage, Variable-Frequency (VVVF)?

AC control system uses a rectifier and an inverter to convert AC power to DC power and then to variable-voltage, variable-frequency, three-phase AC current. This current controls a standard AC motor, which operates at the speed corresponding to the frequency of the input. The result is very accurate and efficient speed control. The VVVF control system does not have the disadvantages of the thyristor system and is energy efficient. It is useful for elevators of all speeds and travel distances, and maintenance is minimized due to its solid-state design.

What is Synthetic Gypsum?

Accounts for 7% of the industry's total use of gypsum. A by-product of various manufacturing, industrial, and chemical processes. The main source of synthetic gypsum in North America is flue-gas desulfurization - a process used by power plants to remove polluting gases from their stacks and thus reduce harmful emissions.

A 1 sq in opening in a 100 sq ft gypsum board partition can transmit how much sound?

Almost as much sound as if the entire partition did not exist.

What is an Energy Transfer Wheel?

Also called (enthalpy heat exchanger) transfer heat between two airstreams through the use of a heat exchanger wheel. Air passes through small openings in the wheel; these openings are impregnated with lithium chloride or another substance that absorbs moisture and transfers it to the other airstream.

What is an induction system?

An air-water system where air is supplied under great pressure and velocity. The air passes through fan coils heated or cooled by the water from either the boiler or chiller.

What is the comfortable range of both temperature and humidity? What is the tolerable range?

Comfortable: 69° F - 80° F and 30% - 65% Tolerable: 60°F - 85° F and 20% - 70%

A tree's caliper size refers to what? A. Height of tree trunk measured from ground to the underside of the canopy B. Diameter of a tree trunk C. Radius of a tree trunk D. Circumference of a tree trunk

B

What are the differences between common path of egress and exit access travel distance?

Common path of egress travel is the portion of the exit access travel distance measured from the most remote point within a story to the point where the occupants have separate access to two exits or exit access doorways and can make a choice about which direction to go. Exit access travel distance encompasses the common path of egress but extends to an exit.

What does the term honeycombing mean when speaking of concrete pours?

It is the unfortunate formation of air pockets.

What is the formula for calculating Noise Reduction if you increased the original total room absorption to an even greater total room absorption?

NR = 10log (Aorg / Anew)

What the hell is Noise Criteria and Preferred Noise Criteria?

Noise Criteria is a chart of decibels associated with background noises for different types of spaces provided in the 8 octave bands. Preferred Noise Criteria provides even lower decibels as some noises conforming to NC still are too loud for comfort at some frequencies.

What is the formula for calculating a sound's power?

Power, I~ is the quantity of acoustical energy as measured in watts. In free space, a point source emits waves in all directions equally, so the sound intensity, I, at a given point at a distance of r from the source is equal to the power divided by the area of a sphere of radius , as shown by Eq. 19.2. I = P / 4πr^2

What are the four stages of water treatment?

Pretreatment - remove suspended matter and large particles. Filtration - passes the water through thin membranes, sand, and other materials that decompose organic material and remove smaller particles. Demineralization - removes dissolved solids and the chemicals that cause hard water. Disinfection - destroys microorganisms that can cause disease in humans. There are several methods used to disinfect water. The EPA Surface Water Treatment Rule (SWTR) requires disinfection of water supply systems that get their water either from surface water or from groundwater under the direct influence of surface water.

What is case hardening?

Produces a hard-surface steel over a relatively softer core.

What is the difference between Recuperative Boilers, Economizer Cycles, and Energy Recovery Ventilators?

Recuperative Boilers recover heat from expelled flue gases from the boiler. Economizer cycles incorporate outside air into the loop to supplement heating or cooling depending on the needs of the building versus the outside temperature. Energy Recovery Ventilators (also called air-to-air exchangers) - reclaim waste energy from the exhaust air stream and use it to condition incoming fresh air.

What is a regenerative drive for an elevator?

Regenerative drives generate electricity when a heavily loaded car travels in the down direction (when the counter weight outweighs the car).

What is a thermosiphon system?

Relies on the natural movement of heated water to circulate the water in a passive, open-loop solar water heating system. This system is also simple, but the storage tanks must be located above the collectors, and the piping must be kept simple to minimize pipe friction. To address the problem of freezing, a variation of this method uses a closed-loop system with antifreeze fluid.

What is R-4 Occupancy Group?

Residential assisted living where number of occupants >5 but <16

What are the first considerations in selecting an HVAC system for a building? A. Kind of control required B. Climatic zone C. Ease of integration of the mechanical system with other building systems D. Anticipated occupancy of the building

D The first consideration when selecting an HVAC system for a building should be anticipated occupancy of the building. A warehouse will have very different heating and cooling requirements than a hospital.

What is R-2 Occupancy Group?

Residential with 3 or more units

What are the three classification of stone shaping?

Rubble - little or no shaping Squared Stone - Stone with slightly shaped edges resulting in vertical joints Ashlar - Highly shaped stone - also called cut stone.

A building that serves as an essential facility would fall under what occupancy category? A. I B. II C. III D. IV E. V

D The occupancy categories range from I (low hazard to human life in event of failure) to IV (an essential facility). An example of a I hazard would be small storage facilities. IV would be hospitals. IV is the correct answer.

How do you calculate the total sensible heat in a building by occupant load?

Each occupant is estimated to release about 225 Btu/hr. Total sensible heat is estimated by multiplying the number of occupants by 225 Btu/hr.

What is Occupancy Group E?

Educational by > 5 people through 12th grade

What are Supplementary Cementitious Materials (SCMs)?

SCMs are added to concrete as part of the total cementitious system and also impart other desirable qualities to a concrete mix. Unlike admixtures, SCMs have cementing properties similar to portland cement and are often used as a partial replacement for portland cement. Fly Ash, Ground-Granulated Blast-Furnace Slag, Silica Fume, & Pozzolan are a few examples.

What is Transition-Metal Hydride Electrochromics?

Glazing material that changes from transparent to reflective. These products are based on coatings of nickel-magnesium instead of oxides used in other electrochromic materials.

What are the three facing grades of hollowed brick?

HBS - general use where a wide range of color and variation in size are acceptable or required. HBX - used when a high degree of mechanical perfection, narrow color range, and minimal variation in size are required. HBA - is nonuniform in color, size, and texture. *Facing brick is only available in SW and MW weathering grades.

What is the difference between sapwood and heartwood and why is it important to differentiate between the two?

Heartwood comes from the center of the tree while sapwood comes from the perimeter. In some species like redwood, the heartwood is much more suitable to resist decay than sapwood. There is also a significant color variance between the two.

What is a deadband?

In a building control system, the range of temperatures within which neither heating nor cooling is needed.

What are the six categories of Seismic Design Category which is defined by the IBC?

Seismic Design Category is determined by three factors: the soil conditions where the structure is located, the risk (or occupancy) category, and the geographical location of the structure. A. Hard Rock B. Rock C. Very Dense Soil and Soft Rock D. Stiff Soil (used as a default when soil conditions are unknown) E. Soft Soil F. Special Soil

What does it mean when glass is labeled "D-H-W-90"?

this would mean that the glazing is appropriate for use in a door or as a wall, meets the hose stream requirements, and has been tested for a 90 minute rating. -D means the glazing is for use in doors and has been tested in accordance with NFPA 252. -H means the glazing meets the hose stream test. -T means the glazing meets the temperature rise requirements.

What is an extract air window?

uses a double-paned insulated glass unit over which another pane of glass is placed on the inside of the building. Air is drawn up between the interior pane and the main window unit and is extracted into the return air system. This warms the glass in winter and cools it in summer to maintain a comfortable radiant temperature and eliminate the need for a separate perimeter heating system.

What is the difference between "yard lumber" and "factory and shop lumber"?

yard lumber is used for rough framing and structural purposes. factory and shop lumber is used for making door frames, windows, and finish items.

What is I-4 Occupancy Group?

daycare for > 5 adults or infants (<2.5 years)

What does F40T12WW/RS mean?

describes a fluorescent lamp (F), 40 W (40), tubular (I), 2/ in diameter (12), warm white color (W'~(O, with a rapid start (RS) circuit.

What are Ground-Couple Heat Exchangers?

heat or cool outside air by circulating it through pipes buried in the ground. In the summer, the air can be used directly if the outdoor air temperature is higher than the ground temperature. In the winter, the system can preheat air for an energy recovery ventilator (to prevent frosting~ or for a standard fan-coil heating unit. Ground-coupled heat exchangers are typically suitable only for low-rise buildings. Their disadvantage is that long runs of pipes are needed for efficient operation. Because the air is forced through the pipes, the energy saved by using such a system must outweigh the energy needed to run the fans. An alternative to this type of system that takes advantage of geothermal energy is the ground source heat pump.

What is laitance?

his is a chalky surface deposit of low-strength concrete. If additional concrete will be poured on top of an existing pour, the laitance must be removed in order for the new concrete to bond.

The equation q = UAΔT represents what relationship?

q is the total heat loss through one unit of area of building material. A is the area being evaluated. ΔT is the temperature differential between outside and inside.

What are the four components of paint?

the binder, pigments, liquid, and additives. The binder and pigments together are often called the solids.

What is Evaporative Cooling?

Water is dropped over fins or pads as warm air is drawn in through a fan. This water then evaporates taking heat along with it. The cooled air is then used to cool spaces inside the building. **This process only works in arid-dry climates otherwise evaporation will not take place in a highly humid environment.

What is Autoclaved Aerated Concrete (AAC)?

a lightweight, precast concrete made with aluminum powder as an extra ingredient. This type of concrete is hardened in molds and cured in an autoclave, which is a steamfilled pressure chamber. The concrete is formed into blocks, typically 10 in by 25 in, in thickness of 4 in, 8 in, and 10 in.

What is Unit Multivoltage (UMV)?

also called the Ward-Leonard control system, uses an alternating current (AC) motor to operate a direct current (DC) generator. The DC current from the genera tor is then varied to run a DC traction motor. Before the advent of electronic motor control in the 1980s, this was the only way to obtain the precise motor speeds needed for smooth elevator operation. However, the system is noisy, requires three machines, and is subject to high thermal losses and low overall efficiency. Many UMV control systems are still in use, but other systems have become more com mon in new elevators.

What should the density of an exterior sound barrier be?

5 lbm/ft² - any denser will not affect the sound attenuation significantly

What percentage of materials and products of a project need to be renewable in order to receive LEED credit?

5%

What are the nominal dimensions of an 8 x 8 x 16 CMU block?

7 5/8" x 7 5/8" x 15 5/8" **The actual dimensions of the CMU are 3/8" less than the nominal dimensions to allow for mortar joints.

How much of the concrete's strength percentage-wise does it gain within the first week of curing?

70%

What is a Kelly Ball Test?

Also known as the ball penetration test, a hemispheric mass of steel with a calibrated stem is dropped onto a slab of freshly laid concrete. The amount of penetration of the ball into the concrete is measured and compared to one-half the values of the slump test.

What is the backset?

It is the distance from the edge of a door to the centerline of a doorknob or pivot of a lever handle.

What is the Brewster Color System?

Also known as the Prang Color System, it is the familiar color wheel that organizes color pigments by their relationship with the three primary colors of red, blue, and yellow.

What is the difference between Solar Heat Gain Coefficient and Shading Coefficient? (SHGC / SC)

SHGC includes the effects of the frame, glass spacer, and other parts of the window assembly while SC is just the glass itself.

What is the process of tempering metal?

Similar to quenching but does not involve rapid cooling. it is also used to improve strength and workability of steel (submersion into water).

What is the difference between flying forms and slipping forms?

Slipping forms move continuously at 6in to 12in per hour to form tunnels or skyscraper cores. Flying forms are used to cast an entire section then it moves up all at once for the section above.

What is Surcharging?

Surcharging is the preloading of the ground with fill material to cause consolidation and settlement of the underlying soil before building. Once the required settlement has taken place, the fill is removed and construction begins. Although suitable for large areas, the time and cost required for sufficient settlement often preclude this method of soil improvement.

How in the world do you calculate for the total transmission loss in a wall of multiple materials?

TL = 10log A / ∑tS You can find the t value of a material if you know the transmission loss of a material. t = 10 ^ -(TL/10)

What is Visual Comfort Probability (VCP)?

The visual comfort probability (VCP) is a metric used in evaluating the problem of direct glare. The VCP is the percentage of normal observers who may be expected to experience visual comfort in a particular environment with a particular lighting situation. Many manufacturers publish VCP ratings for their light fixtures when the fixture is used under defined conditions.

A sprinklered office with 157 occupant load requires the minimum width for the means of egress to be what?

(157 persons)(0.2 in/person) =31.4 in However, IBC also specifies a minimum width of 44 in for a corridor that serves an occupant load greater than 50. The larger value must be used, so the minimum width is 44in.

A wall with 0.1% open area (from cracks, holes, undercut door, etc.) will have a maximum transmission loss of about _______.

30 dB

What is the maximum allowed permeance allowed for an air barrier? (in accordance with ASTM E2178)

0.004 cfm/ft² at a pressure difference of 1.57 lbf/ft² This value is approximately equal to the permeance of a sheet of 1/2 in unpainted gypsum.

How is Noise Reduction calculated and what does it represent?

(NR) is the arithmetic difference (in decibels) between the intensity levels in two rooms separated by a barrier of a given transmission loss. Noise reduction is dependent on the transmission loss of the barrier, the area of the barrier, and the absorption of the surfaces in the receiving room. NR = TL + 10 log (A / S) TL = Transmission loss A = Absorption S = area of barrier

What is the stack effect?

(chimney effect) is a difference in pressure between the top and bottom of a building due to a temperature differential.

What is the speed of sound at sea level through air?

1130 ft/sec

What is Concrete's typical pH level?

12.0 to 13.3

How much square footage should be used to estimate one parking space of a car for a garage?

400 ft²

What are the dimensions of a standard brick?

3 5/8in thick, 2 1/4in high, and 7 5/8 in long **With a mortar joint of ~ in, this gives a modular size of 4 in thick and 8 in long. Three courses equal 8 in, the same as a standard concrete block course.

How much noise reduction results from doubling the amount of absorption in a room?

3 dB

How high can a handrailing be above the nosing of a tread?

34-38"

How low can light fixtures or furnishings project into a corridor that is apart of a means of egress?

4" into an area that is 27" to 80" above the walking surface.

How much surface area should allow for natural ventilation to a floor? (percentage of floor area)

4% When a room or space without outside openings is ventilated through an adjoining room, the opening to the adjoining room must have an unobstructed area of at least 8° o of the floor area of the interior room or space, and not less than 25 ft2. In this case, the minimum area that can be opened to the outdoors is calculated based on the total floor area being ventilated.

How far can a hand rail project into a corridor apart of a means of an egress?

4.5"

What are tolerable wind speeds? (range)

50 - 200 ft/min

A hairline crack will decrease a partition's transmission loss by about _____.

6 dB

An ADA ramp slope is 1:12. What is that slope as a percentage? Round to the nearest hundredth.

8.33 1/12 = x/100 0.083*100 = x x = 8.33

What is I-3 Occupancy Group?

> 5 people restrained

What is I-1 Occupancy Group?

>16 ambulatory people on 24 hour basis assisted living, group home

What diagram consists of the combined force polygons for all the truss joints? A. Free body diagram B. Poly diagram C. Stress diagram D. Vector diagram

A A stress diagram consists of the combined force polygons for all the truss joints. In this diagram, each space between forces and each triangular area is given a label. Each member is then able to correspond to the truss. In the stress diagram, the forces at each joint are plotted to scale. From the scale diagram, forces can be determined for members.

A client's program proposes an art gallery on the main floor and wine tasting in the basement. They have expressed concern about the sound of heels filtering into the wine tasting area from the floor above. A material with an IIC rating of which of the following would be most appropriate to specify? A. 55 B. .1 C. A D. D

A Impact insulation class (IIC) ratings are numerically based. The higher the number the better the assembly at resisting sound transmission.

How do intumescent materials respond to heat? A. Rapid expansion B. Rapid shrinking C. Rapid cooling D. Removing oxygen from the air

A Intumescent coatings are often applied to steel to give it fire-resistant. Intumescent materials rapidly expand when exposed to heat creating a sacrificial layer.

What is silica fume?

A Supplementary Cementitious Material SCM that is collected by filtering the smoke created during the production of silicon and ferrosilicon metals. It consists mostly of particles of silicon dioxide about 1/100 the size of cement grains. Silica fume is also available in liquid form. It is added to concrete in a proportion of 7% to l0% by weight of the cement. Silica flame decreases permeability, increases compressive strength, improves abrasion resistance, and reduces bleeding.

What is Monel Metal?

A combination of copper and nickel with small amounts of other elements. it is highly resistant to corrosion and easily worked. Used primarily for roofing.

Select which items are usually present in the geotechnical engineer's subsurface report. Select all that apply. A. Results of field tests B. Results of laboratory tests C. Recommended types of foundations D. Soil Sieve Analysis E. Saturation point (water table) F. Recommended treatment for contaminated soil

A, B, C, E The geotechnical engineer will conduct field and laboratory tests to determine the soils and their characteristics. They provide this information in their report. Based on their findings, they can usually determine the level of the water table and they will be able to recommend a foundation type. The structural engineer may or may not use this recommendation. A soil sieve analysis is not typically a part of what a soils engineer provides. If contaminated soils are found, conditions need to be assessed by a soil remediation expert.

What kind of forces are resisted by moment connections between beams and columns? A. Gravity B. Lateral C. Temperature D. Dead

B Lateral forces such as wind or seismic loads are resisted by moment connections.

The Owner is mostly concerned about schedule and the design criteria. Construction costs are secondary to the deadline of project completion. What Contractor compensation would an Owner be most likely to choose? A. Cost-plus-fee with a guaranteed maximum price B. Cost-plus-fee C. Stipulated sum D. Unit price

B The correct answer is cost+fee. Typically an Owner may lean more towards cost+fee with a GMP but in this case, cost is not a concern. It's less likely that the Owner will run into conflicts if they remove the ceiling on how much the Contractor can make. A cost-plus-fixed-percentage contract places the liability on the client and rarely is used unless there is some urgency associated with the need for the facility to be built. It gives the contractor no incentive to bring the project in on budget. A cost-plus-fixed-fee contract places the cost liability on the client but also presents the contractor with the incentive to finish the project quickly. A cost-plus-variable-percentage contract provides an incentive for the contractor to bid the job accurately because his percentage will be higher if the job comes in under budget and lower if the job costs more than estimated.

How can heat gain from light fixtures be calculated?

Because I W = 3.41 Btu/hr, heat gains from lighting can be calculated by multiplying the total wattage load of the building's lighting by 3.41. For fluorescent and other discharge lights, the energy used by the ballast must also be included. A general rule is to multiply the Btu hr generated by these types of fixtures by 1.25.

What is the difference between an air-water system and an all-water system?

Besides the obvious, an all air system can expel 100% of the air so that there are no contaminants (hospitals and laboratories).

What is Occupancy Group B?

Business for office or service transactions

Which of the following materials has the most stringent quality control standards overall? A. Brick Masonry B. Concrete C. Steel D. Wood

C

Which type of foundation system is driven into the ground? A. Caisson B. Spread C. Pile D. Mat

C Piles are driven into the ground by a piledriver. Piles may be steel H-shapes, steel pipes, precast concrete, or wood.

Which element is found in the greatest proportion in structural steel? A. Carbon B. Copper C. Iron D. Tin

C Steel is mostly comprised of iron. Steel is an alloy of iron and carbon.

Which lighting types are most poorly suited for application in a cold climate? Check the two that apply. A. LED B. Incandescent C. Fluorescent D. HID

C and D The starting and operation of fluorescent and high-intensity discharge (HID) lighting systems can be affected by abnormal temperatures. In low-temperature environments such as refrigerated areas, fluorescent lamps can experience adverse performance such as "fluttering," reduced light output and difficulty starting by the ballast. In such applications, enclosed fixtures, special "cold weather" fluorescent lamps and/or jacketed lamps can help ensure reliable performance; in addition, fixtures can be located a suitable distance from cold-air source units. Cold weather ballasts that can start the lamps at temperatures below 0°F should be specified for applications where the ambient temperature is expected to be below 50°F. Cold weather electronic and magnetic ballasts can be specified for HO and VHO lamps that can start them at temperatures as low as -20°F.

How can the health risks of asbestos found in building renovation be avoided? Check the two that apply. A. Heat-drying B. Adequate cross-ventilation C. Abatement D. Vaccuming E. Encapsulation

C and E If asbestos is discovered during construction a specialist should be hired to assess the situation. The specialist will most likely recommend abatement (removing) the asbestos or encapsulating (sealing) it. Asbestos is only problematic when it becomes airborne.

What are the types of loudspeaker systems? Check the two that apply. A. Direct B. Indirect C. Distributed D. Static E. Central

C and E The two types of loudspeaker systems are central and distributed. A central system has an array of directional high-frequency units combined with less directional low-frequency units placed above and slightly in from of the sound source. A distributed system consists of low-level loudspeakers located overhead throughout the space.

What causes hardness in water?

Calcium and Magnesium

What is the difference between Design Cooling Load Factor and Design Equivalent Temperature Difference?

Calculating for heat gain through a building envelope is similar to calculating for heat loss using the overall coefficient of heat transmission and the area of the building assembly.. BUT the temperature difference is NOT used. Instead the Design Equivalent Temperature Difference (DETD) must be used. DETD takes into account the air temperature differences, effects of the sun, thermal mass storage effects of materials, colors of finishes exposed to the sun, and daily temperature range. Design Cooling Load Factor (DCLF) is used to calculate heat gain through glazing by multiplying the area of glazing by the factor.

What are the main differences between Absorption Cooling and Compressive Refrigeration?

Compressive Refrigeration uses a refrigerant to extract heat through an evaporator and releases it through the condenser. The refrigerant changes state through this process with the utilization of a compressor. (four parts, compressor, expansion valve, condenser, & evaporator). Absorption Cooling is less efficient than compressive refrigeration but is often used when waste heat is available. This system has four major chambers (condenser, evaporator, generator, and absorber). The system uses water and lithium bromide since the two have a strong attraction to each other.

How "long" is a long span?

Considered to be over 60 ft.

What is the difference between control joints, construction joints, expansion joints, and isolation joints?

Control joints are just a depression in the concrete that allow cracks to run along the depression. Construction joints are simply the delineation of one pour from the other. Rebar extends into the new pour to maintain structural continuity. Prefabricated waterstops are inserted into the first pour to prevent water leakage. Expansion joints are complicated assemblies that allows concrete to contract or expand independently on both x and y directions. Isolation joints allow movement as well but are not as complicated as expansion joints. They typically involve a premolded joint material. These usually occur between columns and slabs.

What is the composition of copper, bronze, and brass?

Copper is composed of bonze and brass. Bronze is composed of an alloy of copper and tin. Brass is composed of an alloy of copper and zinc.

Which of the following is NOT an ingredient in portland cement? A. Alumina B. Iron oxide C. Limestone D. Silicon

D To create portland cement, materials that contain appropriate amounts of calcium compounds, silica, alumina and iron oxide are crushed and screened and placed in a rotating cement kiln. Ingredients used in this process are typically materials such as limestone, marl, shale, iron ore, clay, and fly ash. Silica is a common oxide form of silicon. Silicon is commercially prepared using silica in an electric arc furnace. Both silica and silicon have lattice structures. But silica differs from silicon due to the presence of silicon-oxygen covalent bonding. This accounts for all the different properties between two.

Which of the following is NOT an ingredient in portland cement? A. Alumina B. Iron oxide C. Limestone D. Silicon

D To create portland cement, materials that contain appropriate amounts of calcium compounds, silica, alumina and iron oxide are crushed and screened and placed in a rotating cement kiln. Ingredients used in this process are typically materials such as limestone, marl, shale, iron ore, clay, and fly ash. Silica is a common oxide form of silicon. Silicon is commercially prepared using silica in an electric arc furnace. Both silica and silicon have lattice structures. But silica differs from silicon due to the presence of silicon-oxygen covalent bonding. This accounts for all the different properties of the two.

Typically, what percentage of concrete's volume is taken up by aggregates? A. About 10% B. About 25% C. About 50% D. About 75%

D Typically about 75% of concrete's volume is taken up by aggregates. Cement is expensive so the more space can be taken up by aggregates, the more economical the mix.

Which of the following would be most appropriate for dampproofing on above grade concrete wall with a moderately rough surface? A. cementitious coating B. bituminous coating C. synthetic rubber D. silicone coating

D Which of the following would be most appropriate for dampproofing on above grade concrete wall with a moderately rough surface?

A one-story building with a total area of 10,000 sq ft is divided into a 7000 sq ft restaurant (A2 occupancy) and a 3000 sq ft office (B occupancy). the building is of type V-B construction and does not have a sprinkler system. The open space around the building gives it a 25% allowance for area increase. Does this building comply with IBC maximum area requirements? NS-A2 = 6000 sq ft NS-B = 9000 sq ft

Determine the allowable are for both the A2 occupancy and B occupancy A2 = At + (NS)If = 6000 + (6000)(0.25) =7500 B = At + (NS)If = 9000 + (9000)(0.25) =11,250 sum = A2 actual / A2 allowable + B actual / B allowable 7000 / 7500 + 3000 / 11,250 = 1.2 [must be < or = 1] The sum of the ratios is greater than 1, so the building as configured is not permitted Either the building needs to be sprinklered, or a higher construction type must be used.

How far can a door project into a corridor that is a part of a means of egress?

Doors opening into a means of egress can project 7 in

What is a clo?

Equal to the thermal insulation given by the typical business suit, or about 0.15 per pound of clothing.

What is Equivalent Thickness of a CMU wall?

Equivalent thickness is the thickness of a CMU wall if it were recast with no hollow cores. *Fire code usually provides the required equivalent thickness for the exterior wall.

What is F-2 Building Occupancy?

Factory low hazard

What is F-1 Occupancy Group?

Factory moderate hazard

What is the difference between High Pressure Decorative Laminates and Thermoset Decorative Paneling?

High pressure Decorative Laminate (HPDL) is a thin sheet material made by impregnating several layers of kraft paper with phenolic resins and overlaying the paper with a patterned or colored sheet and a layer of melamine resin. The entire assembly is placed in a hot press under high pressure where the various layers fuse together. Plastic laminates are used for countertops, wall paneling, cabinets, shelving, and furniture. Thermoset Decorative Paneling (also called low-pressure laminates) are made by pressing a decorative overlay from a thermoset polyester or melamine resin-impregnated saturated sheet onto a cellulosic substrate such as particleboard or medium-density fiberboard. The cost is usually lower but the finishes are limited and they are not as durable.

What is Self-Consolidating Concrete?

Self-consolidating concrete (SCC) is a concrete mixture that can be placed by means of its own weight with out the use of vibration. SCC is made possible with the use of a superplasticizer admixture called a poly carboxylate polymer. Because no vibration is needed, SCC placement finishes faster, requires less labor, and increases productivity. This type of concrete flows easily around dense reinforcement and provides a smoother, more uniform surface than standard concrete, so less time is required to make minor cosmetic repairs. Because SCC develops strength faster than conventional concrete, forms can be stripped sooner.

What is the difference between sensible heat and latent heat?

Sensible heat is the transfer of heat through thermodynamics while Latent heat is the release or absorption of heat through change of state.

What is the difference between Sick Building Syndrome / Building Related Illness / Multiple Chemical Sensitivity?

Sick Building Syndrome (SBS) - a condition in which building occupants experience a variety of health related symptoms that cannot be directly linked to any particular cause. Building-Related Illness (BRI) describes a condition in which the health related symptom or symptoms of a building's occupants are identified and can be directly attributed to specific building contaminants. Multiple Chemical Sensitivity (MCS) is a condition induced by exposure to VOCs or other chemicals. People with MCS may develop acute, long-term sensitivity and show symptoms each time they are exposed to the chemicals.

What are the basic metals in stainless steel?

Steel is basically carbon and iron while chromium and nickel are added to give it corrosion resistant properties or stainless steel.

What is S Occupancy Group?

Storage

What does it mean when you say a hinge is 4 X 4 1/2?

You are saying the hinge is 4 inches long and 4.5" in width when fully open.

What is British Thermal Unit?

The amount of heat required to raise the temperature of 1 lbm water by 1°F

What are panel points in regards to trusses?

The are the points where chords meet the top member of a truss.

What does the equation q = V(1.08)ΔT represent?

The heat loss through air infiltration (through cracks and seals rather than through transmission). V = volumetric flow rate, V, of air infiltration 1.08 Btu-min/ft^3 - F°-hr accounts for the specific heat of air, that is, the amount of heat that air at a certain density can hold. ΔT = temperature differential between outside and inside

What is the Hygrometer Test?

The hygrometer test (sometimes called the relative humidity test) determines the moisture emission by measuring the relative humidity (RH) of the atmosphere confined adjacent to the concrete foot In this test, a pocket of air is trapped below a vapor impermeable box, and a probe in the device measures the RH. Test standards recommend that moisture-sensitive flooring not be installed unless the RH is 75 % or less.

What is the difference between ferrous and nonferrous?

There are two major classifications of metals: ferrous and nonferrous. Ferrous metals are those that con tain a substantial amount of iron; nonJèrrous metals are those that do not. The primary types of ferrous metals used in the construction industry include iron, steel, stainless steel, and other special steel alloys. Nonferrous metals do not include iron. Aluminum, copper, bronze, and brass.

What is U Occupancy Group?

Utility

What are Insulating Concrete Forms (ICFs)?

When Polystyrene Foam is used as the formwork for concrete and remains after curing. Finishes are applied on top and greatly speeds up construction. This method is usually only used in small construction or residential projects especially for the basement.

What is a heat pipe?

a self-contained device that transfers sensible heat energy from hot exhaust air to cool outdoor air. As the hot exhaust air passes over the heat pipe, it vaporizes a refrigerant inside the pipe, which then moves to a section of the pipe that is exposed to cool incoming air. As the refrigerant condenses, it releases heat to the incoming air, warming it. A wick material inside the pipe then carries the refrigerant back to the hot side through capillary action. For heat pipes to work, the incoming and outgoing airstreams must be adjacent.

What is Lift-Slab Construction?

a technique for multistory construction in which entire floor sections are cast on the ground, one on top of another around pre erected columns. The slabs are poured with a bond breaker between successive pours. Once cured, the slabs are lifted into place with jacks attached to the columns. The slabs are connected to the columns with weld plates. This type of construction minimizes the amount of formwork required and generally reduces total construction time.

This type of heating/cooling system combines a perimeter hot water heating pipe with an overhead air-handling system. A. Air-Air heat pump B. Cooling coils added to warm air furnace C. Hydronic & coils D. All water system

C A hydronic & coils system combines a perimeter hot water heating pipe with an overhead air-handling system.

What is the term for lengthwise grain separation in wood between or through growth rings? A. Check B. Pitch Pocket C. Shake D. Split

C A lengthwise grain separation in wood between or through growth rings is termed a shake. A check is a lengthwise grain separation that occurs through the seasoning process.

If a Class II wet standpipe system is installed, all portions of the building must be within how many feet of a connection? A. 65 ft B. 100 ft C. 130 ft D. 150 ft

C According to the IBC 2012 §905.5, Class II standpipe hose connections shall be accessible and located so that all portions of the building are within 30 feet or a nozzle attached to 100 feet of hose.

__________ is the total energy required to extract, produce, fabricate, and deliver a material to a job site? A. Allocated energy B. Anticipated energy C. Embodied energy D. Pre-consumer energy E. Life-cycle cost

C Embodied energy is the total energy required to extract, produce, fabricate, and deliver a material to a job site.

Fluorescent bulbs are primarily filled with what kind of gas? A. Halogen B. Hydrogen C. Mercury D. Nitrogen E. Fluorescent lights aren't filled with gas

C Fluorescent bulbs produce light when an electric arc passes between cathodes to excite mercury and other gases producing radiant energy, which is then converted to visible light by a phosphor coating.

Your client owns an authentic 1-story slab on grade Mexican restaurant. On hot/humid days, condensate develops on the smooth floor tile and there is concern that at some point, someone might slip. What is the most economical strategy to remedy this situation? A. Replace the smooth tile with honed tile B. Install insulated windows C. Install a dehumidifier D. Place insulation with an integrated vapor barrier beneath the tile floor

C In this situation, it is likely that the hot/humid air is condensing on the cool surface of the tile. The most economical strategy to try first would be to install a dehumidifier to remove the moisture from the air.

During demolition the contractor uncovers asbestos insulation. The Owner directs the contractor to encapsulate the asbestos. What is the best action for the Architect to take? A. Support the contractor and offer best practices and methods to encapsulate the asbestos B. Recommend the Owner leave the asbestos untouched and purchase a new property C. Recommend the Owner hire an asbestos inspector to determine a method of remediation D. Suggest the contractor hire a specialized asbestos subcontractor to encapsulate the asbestos materials

C It is the Architect's responsibility to protect the health, safety, and welfare of the general public while also having a responsibility to the Owner and the profession. All of these responsibilities can sometimes be hard to manage. In this particular scenario it is the Architect's responsibility to ensure that the method for remediating the asbestos is the most appropriate. The Architect, Owner, and Contractor cannot make that call on their own. An accredited asbestos inspector should be brought in to test, inspect, and recommend an appropriate method for remediation. Hiring an asbestos subcontractor may be the best route for the contractor to take, but a proper inspection is required beforehand to inform the asbestos subtractor of the method and extend to which the asbestos is to be removed.

Which of the following types of wood floors would be the LEAST appropriate for a commercial office? A. block B. parquet C. resilient D. strip

C Resilient wood floors are commonly used for theater stages, dance floors, and gymnasiums. They pro vide extra bounce and resiliency for these types of uses.

Which medium transmits sound the fastest? A. concrete B. air C. aluminum D. water

C Sound transmission is related to density. The more dense something is the more easily it transmits sound. Air - 1,139 ft/sec Water - 4,625 ft/sec Wood - 10,825 ft/sec Brick - 11,800 ft/sec Concrete - 12,100 ft/sec Steel - 16,000 ft/sec Glass - 16,400 ft/sec Aluminum - 19,000 ft/sec ** Sound travels faster through closely packed molecules but will lose significant energy when transferring medias (air to wall for example)

What is the maximum travel distance in an assembly use building that is not equipped with an automatic sprinkler system? A. 75' B. 100' C. 200' D. 250'

C The longest exit travel distance for assembly, factory, mercantile, residential.

If the calculated deflection of a steel beam too high, it's necessary to select a different beam section with a greater ___________. A. Area B. Modulus of elasticity C. Moment of inertia D. Section Modulus

C The moment of inertia is directly proportional to the amount of deflection. If the deflection is high, the moment of inertia must change.

What is the standard AC frequency in the United States? A. 40 Hz B. 50 Hz C. 60 Hz D. 100 Hz

C The standard AC frequency in the United States is 60 Hz. Most other countries use 50 Hz.

What are the three facing grades of brick?

FBS - general use where a wide range of color and variation in size are acceptable or required. FBX - used when a high degree of mechanical perfection, narrow color range, and minimal variation in size are required. FBA - is nonuniform in color, size, and texture. *Facing brick is only available in SW and MW weathering grades.

What is the maximum height to thickness ratio a single-wythe wall can reach unsupported? What about a hollow wall?

20:1 18:1

What is the Calcium Chloride Test?

(sometimes called the moisture dome test) is one of the most common tests for moisture in concrete because it is inexpensive and easy to complete. It gives results in the same form that many flooring manufacturers use to determine if their products can be successfully installed. This test is conducted by placing a standard mass of calcium chloride below a plastic cover and sealing the cover to the concrete floor After 60 to 72 hours, the calcium chloride is weighed and compare with its pretest weight. Through a mathematical formula, the amount of moisture the calcium chloride absorbed is converted to the standard measure of pounds per 1000 ft2 per 24-hour period. One test should be conducted for every 500 ft2 to 1000 ft2 of slab area.

What is included in a contractor's bid?

-material and labor costs - cost of equipment, fees, and services necessary to complete the project. -the contractor's overhead and profit.

Past what sound absorption rating do economics and environment become undesirable?

0.50

What are the 6 climate zone in the United States?

1 - tip of Florida 2 - Florida/Texas 3- South California and The South 4 - Mid-West and Mid-East 5- Nevada, New England, Great Lakes 6- North America/ Canada 7 - Canada 8 - Iceland

What are the 10 major occupancy groups?

1) Assembly 2) Business 3) Educational 4) Factory and Industrial 5) Hazardous 6) Institutional 7) Mercantile 8) Residential 9) Storage 10) Utility

What are the two groups of Indoor Air Contaminants?

1) Chemical Contaminants - volatile organic compounds, inorganic chemicals, tobacco smoke 2) Biological Contaminants - mold, pollen, bacteria, and viruses

What fire rating must an elevator enclosure have if it serves less than or equal to three floors? What fire rating must the enclosure have if it serves more than three floors?

1) Enclosure and openings must have 1 hour 2) Enclosure must have 2-hour fire-rating and openings must have 1.5 hours

What is the maximum length of a corridor with a dead end? (Answer in feet without units Ex: 12)

20 According to section 1018.1 Dead Ends of the IBC Where more than one exit or exit access doorway is required, the exit access shall be arranged such that there are no dead ends in corridors more than 20 feet. The correct answer is 20 feet, which should have been answered as 20.

An office building is being built with Type IIB construction and a sprinkler system. Each floor will have the same area. The allowable area factor increase has been determined to be 25%. What is the maximum allowable area for the building? TYPE IIB SM Allowable area = 69,000 Allowable floors = 4 TYPE IIB NS Allowable area = 23,000 A) 300,000 B) 370,000 C) 460,000 D) 550,000

A Aa = (At + (NS)If)Sa = ((69,000 + 23,000)(0.25))(4) = 299,000 Check the allowable area for an individual floor by using a value of Sa = 1 Aa = (At + (NS)If)Sa = (69,000 + (23,000)(0.25))(1) =74,750

What type of luminaires are strictly intended for decorative lighting? A. Accessory lighting B. Recessed lighting C. Task-ambient lighting D. Torchere lighting

A Accessory lighting is strictly intended to be used as decorative lighting.

An Architect is designing a restaurant. The space can accommodate over 100 occupants. What occupancy group in the building code would this use fall under? A. Assembly B. Mercantile C. Business D. Mixed Use

A According to section 303.3 of the IBC, a restaurant would be classified in assembly group A-2. The A-2 group is intended for assembly uses for food and/or drink consumption. Note that the number of occupants is specified in the question because Exception 303.1.a.1 of the IBC states that an assembly space with an occupant load of less than 50 persons shall be classified as a Group B (business) occupancy.

If wood has a moisture content of 30%, it is generally referred to as being _________. A. green B. wicked C. ripe D. dry

A If wood has a moisture content of 30% or greater it is referred to as being green. This means that is has not had a chance to season or dry yet. Wood with a moisture content less than 20% will not rot if it maintained below that percentage.

A heat pump in a refrigeration loop is used to __________. A. Bring heat into a space B. Remove heat from a space C. Monitor room Temperature D. Remove heat from a condenser

A Refrigeration is simply the displacement of heating. A heat pump can be used in a refrigeration cycle to bring heat into a space.

Designing for which of the below floodplains would require the most stringent requirements? A. 2-year floodplain B. 5-year floodplain C. 10-year floodplain D. 100-year floodplain

A The 2-year floodplain would require the most stringent requirements because a flood is statistically likely to occur once every two years. A 100-year flood is statistically likely to occur only once every 100 years. This does not mean that it cannot occur more frequently than that, even within the same year. However 100-year flood areas are perceived by FEMA (Federal Emergency Management Agency) as moderate to low risk and thus the regulations are less stringent. The amount of rainfall that would have to occur to trigger a 100-year flood would certainly create a more severe flood and hazardous conditions, but because it is statistically less likely to occur, it is lower risk.

What is the new town concept? A. The idea that new communities can be built away from the crowding and ugliness of existing cities B. The idea that new communities should exist symbiotically with preexisting cities C. The idea that new communities should be modeled after ancient cities D. The idea that towns should be created with integrated employment and shopping centers.

A The New Town concept originated in Great Britain in the 1940s. The concept revolved around the idea that new communities should be located outside of the chaos and ugliness of the city. The idea largely failed in England and the United States because it created isolated towns that lacked employment opportunities.

Which of the following is true of the 'bulk plane restriction'? A. It ensures sufficient light and air for neighboring properties B. The plane is delineated clearly and visible from the street C. The plane is flat D. The envelope slopes at 1:2 downward from the center of the property to the property line

A The bulk plane is a series of planes which limit the allowable volume of space the building can occupy. As with the height limitations, the bulk plane requirements are separated into limits for the front of the lot and the rear. The purpose of the bulk plane is to allow adjacent neighbors access to sunlight and to maintain privacy. Because of the sunlight requirement, the planes follow the length of the property and slope inward.

What is the approximate current in a 120 V circuit serving nine 150W downlights? A. 11 A B. 15 A C. 20 A D. 26 A

A The power in the circuit is 9 X 150W — 1350W Rearrange Eq. 17.4 to solve for current, L Because the circuit has only resistive loads (incandescent lights), use a power factor of 1.0. P= VJ(pf) I = P / V (pf) = 1350 W / (120 V)(1.0) = 11.25 A

What is the simplest type of HVAC system? A. Direct Expansion System B. Reheat System C. Single-Duct System D. Variable air volume system

A The simplest type of HVAC system is a direct expansion system. "A direct expansion air conditioning (DX) system uses a refrigerant vapour expansion/compression (RVEC) cycle to directly cool the supply air to an occupied space."

Volatile organic chemicals (VOCs) are primarily composed of carbon and hydrogen. Methane (CH4) is the most abundant VOC in the atmosphere. A. 1) Cement 2) Aggregates 3) Water B. 1) Aggregates 2) Cement 3) Water C. 1) Water 2) Cement 3) Aggregates D. 1) Water 2) Aggregates 3) Cement

A When mixing concrete ingredients are typically added in the following order: 1 - Cement 2 - Aggregates 3 - Water

How does wood's coefficient of thermal expansion compare to that of other common building materials, like steel? A. Very low B. About the same as C. Very high D. Exponentially higher

A Wood's coefficient of thermal expansion is very low. Pine, for example, has a linear temperature expansion coefficient of 5 where steel is around 11.

What are the differences between a 2 pipe, 3 pipe, and four pipe system?

A 2 pipe system either pumps hot or cold water into spaces with the utilization of fan coils to cool or heat the space then uses another pipe as return. A 3 pipe system as separate pipes for hot and cold as supply but only has one pipe for return. A 4 pipe system has separate supply and return pipes for hot and cold.

What is Ground-Granulated Blast Furnace Slag (GGBFS)?

A SCM (supplementary cementitious material) is produced from the material formed from molten slag that is a by-product of iron and steel manufacturing. The material is dried and ground to a fine powder to produce a hydraulic cementing material that is substituted for portland cement in different ratios; a 250 o to 500 o substitution is common. Slag improves workability, decreases the need for water, and increases setting time in concrete, all of which can be a benefit in large pours and during hot weather. It also reduces bleeding, improves resistance to sulfate and chloride attack, and can pre vent damage from ASR. Concrete that contains slag develops strength more slowly than standard concrete, particularly in the first seven days, but it continues to gain strength beyond 28 days and has a higher ultimate strength than standard portland cement concrete.

What is Pozzolan?

A Supplementary Cementitious Material that is a siliceous or aluminosiliceous material that, in finely divided form and in the presence of moisture, reacts chemically with the calcium hydroxide released by the hydration of portland cement to form various cementitious compounds. Pozzolans are used as a partial replacement for portland cement and to decrease permeability, increase strength, and improve resistance to ASR and sulfate attack. Natural pozzolans, which are formed from clay, shale, and other materials with cementitious properties, include calcined shale, calcined clay, and metakaolin. The Romans used volcanic ash, another natural pozzolan, to mix the earliest form of concrete.

What welds are most commonly used in structural connections? Check the two that apply. A. Fillet welds B. Groove welds C. Solid welds D. Chamfer welds

A and B Filet welds and groove welds are most commonly used in structural connections. Chamfer and solid welds are both fictional terms.

Which elements are volatile organic chemicals primarily composed of? Check the two that apply. A. Carbon B. Hydrogen C. Oxygen D. Chlorine E. Helium

A and B Volatile organic chemicals (VOCs) are primarily composed of carbon and hydrogen. Methane (CH4) is the most abundant VOC in the atmosphere.

What are the most common steel structural systems? A. Beam and Girder System B. Closed-Web Steel Joist System C. Open-web steel joist system D. Type A steel framing system

A and C The most common steel structural systems are the beam and girder system and open-web joist system.

There can be considerable variation in natural brick colors. This variation is caused by which of the following? Check the two that apply. A. The clays from which they bricks are made B. The length of time they are stored C. Pigmented admixtures used D. Temperatures at which they are fired E. The surface on which they are stored

A and D

The subgrade under a paving system should be which of the following? Check the two that apply. A. Compacted fill B. Loose fill C. Organic fill D. Backfill E. Undisturbed Soil

A and E The subgrade will ultimately carry the load of the pavement. To carry the load, the soil needs to be compact and therefore should be undisturbed soil or compacted fill. Organic soil is not compact and will decompose over time. Backfill replaces soil that is removed during building construction. It is used to strengthen and support a structure's foundation. Backfill is generally a combination of soil, stones and rocks.

Which of the following are the most common ways a concrete beam fails? Check the two that apply. A. Bending of the concrete B. Excessive malleability of the concrete. C. Trace amounts of carbon in the steel. D. Trace amounts of iron in the steel E. Yielding of the reinforcing steel

A and E The two most common ways a concrete beam fails is by bending of the concrete and yielding of the reinforcing steel.

What is a belt course or a sill course?

A belt course is a continuous course of bricks that run around the entire building (also sometimes called a string course). The course is usually of a different brick to project a strong horizontal line. It is called a sill course if it is right below the window sill.

What is Formaldehyde?

A colorless gas with a pungent odor. It is used in the preparation of resins and adhesives most commonly found in particleboard, wall paneling, furniture, carpet adhesives, and other glues used in the construction and furniture industries. Formaldehyde is designated as a probable human carcinogen and causes irritation of the eyes and respiratory tract.

What is an Air-Entraining Agent?

A concrete admixture that forms tiny, dispersed bubbles. This increases the workability and durability of the concrete and improves its resistance to freezing and thawing cycles. The agent also helps reduce separation of the components as the mix is poured into forms.

What is an actuator?

A device in a building control system that receives commands from a controller and activates a piece of equipment.

What is a drain-down system?

A direct, active system that solves the problem of freezing by automatically draining the water from the collectors when the outside temperature is near freezing. Because water is wasted whenever the system is drained, this method is best for climates with mild winters where the draining process would not occur frequently.

What is creosote?

A distillate of coal tar and is effective protection against insects. It is insoluble in water and is relatively easy to apply. Creosote is used mainly to protect railroad ties, marine timbers, and roadway guard posts. It is not used in building applications.

What is a fire wall?

A fire-resistance-rated wall that is used to separate a single structure into separate construction types. They must extend from the foundation to the roof. If one building collapses then the other portion must be able to withstand the collapse.

What is the difference between a float and a strike-off?

A float looks similar to a janitor mop and is used by pushing and pulling across a wet concrete surface to produce a smooth finish. A Strike-off is just a gangster piece of wood used to finish a surface.

What is a titration test?

A little bit more involved than the typical pH Test, a titration test involves grinding up a portion of the concrete, mixing the resulting powder with demineralized water, and performing laboratory chemical analysis.

What is the difference between Lumens and Footcandles?

A lumen is a measure of the total amount of visible light emitted by a source. A footcandle is the illuminance cast on a surface by a one candela source one foot away.

What is a desiccant?

A material, either liquid or solid, that absorbs water. Commonly used in these systems include silica gel, zeolite, lithium bromide, and monoethylene glycol.

What is the Standard Penetration Test?

A measure of density of granular soils and the consistency of some clays. In this test, a 2 in diameter sampler is driven into the bottom of the borehole by a 140 Ibm hammer falling 30 in. The number of blows, N required to drive the cylinder 12 in is recorded.

What is Xeriscaping?

A method in which drought resistant and low water plants are used so as to conserve water, and eliminate the need for routine watering.

What are the ingredients of glass?

A mixture of silica sand and small amounts of alkaline salts such as lime, potash, and soda.

What is Sisal?

A natural material used for wall and floor coverings. Made from the fibers of the sisal plant. They are durable, low maintenance, and reduce sound reflection and transmission.

What is the pitch, stitch, and pile height of a carpet?

A pitch of a carpet is the number of warp lines of yarn in a 27 in width. The stitch is the number of lengthwise tufts in 1 in. **The higher the pitch and stitch numbers, the denser the carpet will be. The pile height is the height of the fiber from the surface of the backing to the top of the pile. **Generally, shorter and more tightly packed fibers result in a more durable carpet.

What is a plasticizer?

A plasticizer reduces the amount of water needed while maintaining the needed consistency for correct placement and compaction. Reducing the water makes it possible to mix higher strength concrete.

What is a Poured Gypsum Deck?

A poured gypsum deck is used for roofs and is similar to concrete in that a liquid mixture is poured on reinforcing material. In typical gypsum deck construction, purlins support fiber plank or rigid insulation. Wire mesh reinforcing is placed over the insulation and gypsum is poured on the assembly to a minimum depth of 2'/2 in. Gypsum provides a highly fire-resistant roof deck. Precast gypsum planks with tongue-and-groove edges are also available in 2 in and 4 in thicknesses. These planks are reinforced with wire fabric and can span up to 10 ft.

What is a reglet?

A preformed metal shape that is cast into a concrete wall and set flush with the wall to allow the insertion of the edge of flashing and a sealant.

What is the annealing process?

A process in which the metal is reheated and slowly cooled to obtain a more ductile metal, which will have improved its machinability and cold-forming characteristics.

How is a public way defined in regards to a means of egress?

A public way is the destination in the case of a means of egress. It is any street, alley, or similar parcel of land essentially unobstructed from the ground to the sky that is permanently appropriated to the public - has a clear width of 10 ft.

What is the visible spectrum?

A range of visible light from 400 nm (nanometers or 10^9m) for violet light to about 700 nm for red light.

What does a retarder do for a concrete mix?

A retarder slows down the setting time to help reduce the heat of hydration.

What are degree days?

A rough measure of how much heating is needed for human comfort in a particular location over the course of a year. For example, if the 24-hour average for April 8 in Pennsylvania is 36°F, then the number of degree day for that day at that location is 65- 36 = 29. If the average outside temperature is 65°F or above, there are no degree days for that day.

What is a water-loop heat pump system?

A series of heat pumps for different zones of a building. When some zones are in cooling mode and are dumping heat into the loop, other zones are in heating mode and are extracting heat from the loop; therefore, no additional energy has to be added or removed when the heating and cooling modes are balanced. Only when most of the units are in the same mode does the water in the loop have to be cooled or heated with a cooling tower or boiler. Automatic valves at the cooling tower and boiler direct the water as needed.

What is an Accessory Occupancy?

A space or room that is ancillary to a main occupancy but that does not exceed 10% of the floor area of the story in which it is located. The different occupancies do not need to be separated under these conditions. Unless: Group H must be separated from other occupancies. Sleeping units in Group I-1, R-1, R-2, and R-3 must be separated from other units and from accessory occupancies with a fire partition.

What criteria must be met for the following widths of stairs? 36" 44" 48"

A staircase can be less than 44" if the occupant load is less than 50 people.

What is an Environmental Product Declaration (EPD)?

A standardized report giving preset variables of a product to be compared with other products. It gives guidelines for an Life-cycle assessment to be carried out. An Industry-wide EPD can be created analyzing a category of products or a product-specific EPD.

What is is a electrochromic glazing?

A switchable glazing that consists of a multilayered thin film, applied to glass, that can change between opaque and clear or change colors when a burst of low-voltage electrical current is applied.

What is a batch system?

A type of solar water heating that heats water directly in a black painted tank inside a glazed box. This passive system is simple, but it is subject to freezing and nighttime heat loss.

What is a Ton of Refrigeration?

A unit used to describe the capacity of a refrigeration system. A ton of cooling is the cooling effect obtained when 1 ton of 32 degrees F ice melts to water in 24 hours. This is equivalent to 12,000 Btu/hr. In general, the needed capacity of a refrigeration machine in tons can be calculated by dividing the calculated total heat gain in Btu/hr by 12,000.

What is the difference between vapor barrier and vapor retarder?

A vapor barrier is a thin sheet material, generally plastic, designed to prevent water vapor from passing through it. A vapor retarder, on the other hand, only slows the rate of water vapor transmission. A vapor barrier should have a permeance not exceeding 0.04 perm and be at least 10 mils thick.

What is the difference between a conditional use permit and a variance?

A variance is permission to deviate from the zoning regulations. A conditional use permit allows a nonconforming use or other use in the zoning ordinance if the property owner meets certain conditions.

What is Efflorescence?

A white, crystalline deposit of water-soluble salts on the surface of brick masonry. When water seeps into the masonry, it can dissolve soluble salts present in the masonry, backup wall, mortar, or anything in contact with the wall. The dissolved salts are brought to the surface of the brick and appear when the water evaporates. Although unsightly, efflorescence is usually not harmful to the brick.

What type of effects can excess moisture have most on indoor environmental quality? Check the three that apply. A. Growth of bacteria B. Growth of fungi C. Shortness of breath D. Burning eye E. Growth of dust mites

A, B, C Excess moisture can increase the growth of fungi, bacteria, and dust mites. Excess moisture is often created from cooking, washing, and exhaling. It can be mitigated or resolved through exhaust ventilation and dehumidification.

Select all of the regulatory documents that may affect a building's design. A. Building Codes B. Zoning Ordinances C. Historic Districts D. Use code E. FAA F. Watersheds G. US parking code

A, B, C, E, F Building codes and zoning ordinances are a standard part of any architectural project. Depending on the location, regulations for historic districts, watersheds, and the FAA (Federal Aviation Administration) may come into play as well. The "use code" and "US parking code" are fictional.

What are characteristics of a 'wythe'? Check the three that apply. A. Continuous wall section B. May be one masonry unit in thickness C. Segmented D. May be two masonry units in thickness E. More vertical than horizontal

A, B, E A wythe is a continuous vertical section of masonry one unit in thickness. A wythe may be independent of, or interlocked with, the adjoining wythe(s). A single wythe of brick that is not structural in nature is referred to as a veneer. A multiple-wythe masonry wall may be composed of a single type of masonry unit layered to increase its thickness and structural strength, or different masonry units chosen by function, such as an economical concrete block serving a structural purpose and a more expensive brick chosen for its appearance. While a wythe is considered a continuous vertical, there is no requirement for one to be taller than it is wide to maintain its classification. Segments are not a characteristic of wythes.

Which 4 of the below items are land restrictions imposed by local jurisdictions through the zoning ordinance? A. Setbacks B. Covenants C. Building Height & Area D. Accessibility E. Parking F. FAR G. Easements

A, C, E, F Setbacks, Building height & area, Parking, and FAR are all regulated within zoning ordinances. Accessibility is a life/safety concern that is addressed within ADA standards. Covenants & easements are ordinance related but are specific to each site and typically not regulated within the zoning ordinance.

Which areas in renovation of existing construction that are the most likely places where lead might be a concern. Check the three that apply. A. Existing Paint B. Ceiling Insulation C. In drinking water D. Existing Pipes E. Interior Door Frames

A, C, and D Lead is a hazardous material that may be discovered during renovations of older projects. It may be discovered in existing paint, drinking water, or existing pipes or plumbing. Additionally, it may be found in ceramics, soils, dust solders.

What is bookmatching, slipmatching, and random matching?

All three of these methods are ways of arranging veneer on a wall cut from the same tree. Bookmatching is erecting the veneers opposite of each other so that two of the sheets look continuous similar to pages of a book. Slipmatching is simply erecting the veneers in the same direction the entire length of a wall. Random matching is putting the veneers on a wall in no particular orientation.

What is an accelerator in a concrete mix?

An accelerator speeds up the hydration of the cement so that the concrete achieves strength fastest. This allows faster construction and reduces the length of time needed for protection in cold weather.

What is terneplate?

An alloy of 75% lead and 25% tin can be used to plate steel for roofing.

What is the difference between an exit and an exit discharge?

An exit is simply an enclosure or door that leads to the exterior including fire stairs + corridor, which are fully protected. An exit discharge is the portion of the egress system between the termination of an exit and a public way. This may include exterior exit balconies, exterior exit stairways, exit courts, lobbies of multi-story buildings.

What is a drain-back system?

An indirect, active system that uses water as the heat-collector fluid in a solar heating system. The heated water is pumped to a heat exchanger where a coil of domestic hot water is heated. When the control ler senses the temperature is too 10% it turns off the pump, and the collector water drains back into the solar storage tank.

What is an Incidental Use?

Ancillary to a main occupancy and has the same classification as the nearest main occupancy, but poses a greater level of risk than that occupancy. The incidental use must be separated from the main occupancy by a fire barrier, be equipped with an automatic sprinkler system, or both. An incidental use area cannot exceed 10% of the area of the story on which it is located.

What is a Switchable glazing?

Are chromogenic window products that change their characteristics based on particular environmental conditions or through human intervention.

What is the Cylinder Test?

As the concrete is placed, samples are put in cylinder molds that are 6 in diameter and 12 in high, and are moist-cured and tested in the laboratory according to standardized procedures. Plastic cylinders are sometimes used for this test and are 4 in diameter and 8 in high. The compressive strength in pounds per square inch is calculated and compared with the value used in the design of the structure. Cylinders are tested at a specified number of days, normally 7 and 28 days. Seven-day test results are usually about 60 % to 70 % of the 28-day strength.

What is A-3 Occupancy Group?

Assembly for worship, recreation, etc. e.i. libraries, art museums, conference rooms

What is A-1 Occupancy Group?

Assembly with fixed seats for viewing of movie theaters, live performance

What can high alkalinity do to finishes on top of concrete and around what pH level does this happen?

At a pH level of about 9 or 10, most tile adhesives begin to experience problems, although professional-grade adhesives can sometimes be used on a slab that has a pH of 11. Surface alkalinity can be controlled with various proprietary coatings. *8.5 is ideal for application of finishes

What is the most economical grading of aggregates? A Mostly small aggregate B. 50-50 mix of small and large aggregate C. Mostly large aggregate D. All large aggregate

B A 50-50 mix of small and large aggregate is the most economical grading of aggregate because it creates the smallest voids between the aggregates. The smaller the voids the less cement required.

What type of elevator should be specified for a 40-story office building? A. Hydraulic B. Gearless Traction C. Geared Traction D. Electric

B A gearless traction elevator will be an appropriate choice for the office building. Gearless traction elevators can travel at the highest speeds of the types listed, and to accommodate the rush of people entering and exiting the building in the morning, at lunch time, and in the evening, the quick cycles will be a necessity. Gearless traction and geared traction are two different types of electric elevators. Both operate on DC current. The geared traction elevator travels at slower speeds but offers many options for adjusting the speed to suit the building conditions. Hydraulic elevators are lifted by a ram, which must be sunk into the ground the same distance as the height of the elevator's path of travel. Therefore, they are used only in low rise buildings generally less than six stories). They travel much more slowly than electric elevators and are better used for freight or for low-occupancy passenger elevators where speed is not an issue.

Your client would like to install a fireplace for the aesthetic but is concerned about its inefficiency. What material can be specified around the fireplace to to increase its effectiveness? A. Metal Liner B. Masonry Mass C. Foam Insulation D. Wood Framing

B A masonry mass around a fireplace can help the fireplace store and release heat. A metal liner is highly conductive and would heat up and cool down rapidly. Foam insulation is combustible and not recommended in locations where a lot of heat is expected. Wood framing is also combustible and would not have much of an impact on the efficiency of the fireplace.

Of the listed uses, which would require panic hardware at the exits? A. Motel guest room B. Movie Theater C. High-rise office suite D. Industrial manufacturing facility

B A movie theatre would require panic hardware at the exits. Panic hardware is required per the IBC when the occupant load exceeds 50 and the use is educational, assembly, or high hazard.

Which of the following flooring types has the highest resilience? A. asphalt B. cork C. linoleum D. vinyl composition

B Cork is a very resilient material. Its resilience is similar to that of rubber tile. Asphalt tile, which is seldom used, has the lowest resilience. Linoleum and vinyl composition tile have low to moderate resilience.

What is the unit that measures the amount of light striking a surface? A. Candlepower B. Footcandle C. Lumen D. Watt

B Footcandles measure the amount of light striking a surface. Footcandles are a unit of illuminance or illumination, equivalent to the illumination produced by a source of one candle at a distance of one foot and equal to one lumen incident per square foot.

Which of the following is NOT a category of moment-resisting frame? A. IMRF B. LMRF C. OMRF D. SMRF

B IMRF - Intermediate moment-resisting frame OMRF - Ordinary moment-resisting frame SMRF - Steel moment-resisting frame LMRF is not a category of moment-resisting frame

What site would be best suited for a mat slab or pile foundation? A. A steep cliff-like site B. A site with a high water table C. A flat site D. A site near a lake

B Mat and pile foundations would be appropriate solutions where there is a high water table. The main idea behind using these two types of foundations in this condition is so the structural integrity of the foundation is not compromised. Since a mat foundation is shallow, it does not come into contact with the water table. A pile foundation would extend past the water table to be supported below it.

Up to 20% of a building's total energy consumption is consumed by this appliance; and of the items listed it consumes the most energy. Which is it? A. Forced air unit B. Water heater C. Air conditioner D. Furnace

B Most buildings produce hot water in a large tank called a hot water heater. When the stored water drops below a certain temperature, the water heater turns back on and consumes electricity in order to heat the water. Up to 20 percent of all of the energy used in a building can be attributed to the hot water heater. Insulating the water heater with a wrap, turning down the temperature, selecting an Energy Star rated model, or using a solar hot water heater can reduce energy consumed by hot water heaters by up to half.

A conference room and an office are separated by a common wall that is 13 ft long and 9 ft high, with an STC rating of 54. The total absorption of the office has been calculated to be 220 sabins. What is the total noise reduction from the conference room to the office? A. 46 dB B. 57 dB C. 64 dB D. 72 dB

B NR = TL + 10log A / S = 54 dB + 10log (220 sabins / (9 ft)(13 ft)

Which of the following woods must be treated for resistance to decay when used in an exterior application? A. Cedar B. Spruce C. Redwood D. Cypress

B Spruce trees are not inherently resistant to decay, but the heartwood of cedar, redwood, and cypress all possess a natural resistance that makes them a good choice for use in exterior applications such as siding, shingles or shakes, and decking. The outer rings of these species do not possess the same decay-resistant qualities, so it is important to specify that only heartwood may be used when decay resistance is of concern. These woods are considerably more expensive than other species that are more commonly used for construction such as pine, hemlock, fir, and spruce. Species that are not inherently decay resistant must be chemically treated to protect them from rotting when exposed to moisture. Waterborne salts are pressure-impregnated into the wood to prevent decay, often for as long as 30 years. After a waiting period, salt treated wood can be painted or stained.

Fee simple ownership of land is A) Absolute ownership of land by a governmental authority. B) Absolute ownership of land without condition. C) Ownership of land for 99 years. D) Ownership of land for a simple fee.

B The answer is absolute ownership of land without condition. Under fee simple ownership, land can be used or transferred by its owner as he or she pleases.

A wall is 9 ft high and 15 ft long and contains a 7 ft by 3 ft door. The transmission loss of the wall alone is 54 dB. The transmission loss of the door with full perimeter seals is 29 dB. What is the combined transmission loss of the wall and door? A. 25 dB B. 37 dB C. 56 dB D. 83 dB

B The total area of the wall and door combined is A = H * L = (9 ft)(15 ft) = 135 sqft The area of the door is Sdoor = H* L = (7 ft)(3 ft) = 21 sqft The area of the wall alone is Swall = Stotal - Sdoor = 135 sqft - 21 sqft = 144 sqft From the problem statement, the transmission loss for the wall alone is 54 dB. ficient of transmission for the wall alone is t = 10^-(TL/10) = 10^-(54/10) = 10^-5.4 Similarly, the coefficient of transmission for the door is t = 10^-(TL/10) = 10^-(29/10) = 10^-2.9 From Eq. 19.8, the transmission loss for the wall and door together is TL composite = 10log(Atotal/∑tS) = 10log (135 sqft / (10^-5.4)(114 sqft) + (10^-2.9)(21 sqft)

What type of joints would you use to avoid cracks caused by shrinkage in a concrete slab? A. Construction Joints B. Control Joints C. Expansion Joints D. Isolation Joints

B To avoid cracks caused by shrinkage in a concrete slab control joints should be used.

What grade percentage would be noticeable but could accommodate most activities? A. 2% B. 8% C. 13% D. 15%

B site slope between 4% and 10% is noticeable but can accommodate most activities. Anything below 4% is barely noticeable and appears flat. Slopes above 10% may make activities prohibitively difficult.

In which conditions is wood strongest? Check the two that apply. A. Forces perpendicular to grain B. Forces parallel to grain C. Shear parallel grain D. Shear perpendicular to grain

B and D Wood is strongest against forces applied parallel to the grain and when shear is perpendicular to the grain.

A 10% undervoltage would do what to fluorescent lighting? Check the two that apply. A. Stalling B. Reduced output C. Hotter operation D. Reduced life E. Poor start time

B and E Undervoltage occurs when the average voltage of a three-phase power system drops below intended levels, and is sometimes referred to as a brown-out. A 10% undervoltage would cause fluorescent lighting to have a poor start time and reduced output.

What conduit systems are suitable for embedding in concrete? Check the three that apply. A. Armored cable B. Flex Conduit C. IMC Conduit D. Rigid Conduit E. Stripped Cable

B, C, D

A client is looking for a total net assignable square footage. Which of the following are included in this number? Check the three that apply. A. Public restrooms B. Gym C. Lounge D. Mechanical Room E. Lobby space F. Wall thickness

B, C, E The Net Assignable Square Footage of a floor is calculated by adding all the rooms on a floor Areas excluded are: public corridors elevators stairwells mechanical rooms public bathrooms custodial rooms shaft spaces wall thickness

Why is camber often built into a glued laminated beam? Check the three that apply. A. To reduce flexural stress B. To avoid the appearance of sag C. To allow the use of lumber with a lower stress rating D. To eliminate the ponding of water E. To compensate for deflection F. To minimize deflection

B, D, and E Camber is often built into glulam beams: ◦to avoid the appearance of sag ◦to eliminate the ponding of water on top of a sagging beam ◦to compensate for deflection

A small commercial building with a roof area of 10,000 ft² is located in an area that receives an annual rainfall of 20 in. 25% of the water is lost in evaporation, run-off, absorption, and impoundment. The amount of rainwater that can be collected in a year is... A. 9,000 gal B. 15,000 gal C. 90,000 gal D. 150,000 gal

C The total annual rainfall is equal to the catchment area times the average annual rainfall. total rainfall = (20 in/yr)(10,000 ft²) = 200,000 in-ft² Multiply the total annual rainfall by 0.75 to account for losses, and convert to gallons. Available rainwater = (1 - 0.25)(200,000 in-ft²) (0.6 gal / in-ft² ) = 90,000 gal

Which type of loudspeaker system is most appropriate to install in a low-ceiling area? A. Direct B. Indirect C. Distributed D. Static E. Central

C There are only two principal types of loud speaker systems: central and distributed. In locations with low ceiling distributed, or overhead, loudspeakers are most appropriate.

By what means of heat transfer does wind movement increase evaporation? A. Conduction B. Conflation C. Convection D. Radiation

C Through convection wind movement increases evaporation.

Which material has the lowest perm rating? A. 10 mil (0.25) polyethylene B. gypsum wallboard C. 1 mil (0.025) aluminum foil D. exterior oil paint

C Vapor retarders are selected based on their perm ratings. The best vapor retarders have the lowest ratings. This problem is essentially asking which of the listed materials is the best vapor retarder, and the answer is option C, aluminum foil, with a perm rating of zero. Polyethylene is the next best, with a perm rating of 0.03. Three coats of exterior oil paint have a perm rating of 1.6 to 3.0. Gypsum wallboard allows the most moisture to pass through, with a perm rating of 50.

What is Carbon Fiber Concrete?

Carbon fiber concrete uses epoxy-coated carbon fiber mesh in place of standard steel mesh for secondary steel reinforcement. It is used to make precast panels thinner and lighter. Because carbon fiber is non corrosive, less concrete cover is required. The resulting panels require smaller foundations and support structures, reduce transportation costs, and speed the erection process. The carbon fiber is manufactured by extruding industrial-grade carbon into ultrathin fibers. The fibers are bundled together to form pieces resembling yarn, called tows. The tows are laid perpendicular to each other in a grid, with the intersecting tows bound together by a heat cured epoxy resin. The resulting grid, or fabric, is about 0.04 in thick. The various components of the grid can be modified to meet different strength requirements, with a typical grid having nearly seven times the tensile strength of standard steel mesh.

What is grey cast iron and white cast iron?

Cast iron with a low silicon content is called white cast iron and has little use in construction unless it is processed in such a way as to produce malleable iron. Cast iron with a high silicon content is called gr~v cast iron and is used for various types of castings such as plumbing valves, pipes, and hardware.

What is the difference between a check, pitch pocket, shake, split, and a wane?

Check is a separation of the wood fibers occurring across or through the annual growth rings a result of improper seasoning. A pitch pocket is an open area between growth rings that contains resin. A shake is a lengthwise separation of the wood that usually occurs between or through the annual growth rings. A split is similar to a check except that the separation extends completely through a piece of lumber, usually at the ends. A wane is the presence of bark or absence of wood from any cause on the edge or corner of a piece of lumber.

What is the methenamine pill test?

Current federal law requires all carpet to meet the requirements of ASTM Standard D2859 (Standard Test Method for Ignition Characteristics of Finished Textile Floor Covering Materials). In this test, a methenamine tablet is ignited in the center of a carpet sample held in place with a metal plate with an 8 in diameter hole. If the carpet burns to within I in of the metal plate, the sample fails.

Which type of system voltage is most appropriate for a large building with fluorescent lighting? A. 120-V, Single-Phase, 2-wire B. 120/240-V, Single-Phase, 3-wire C. 120/208-V, Single-Phase, 3-wire D. 277/480-V, 3-Phase, 4-wire

D 120-V, Single-Phase, 2-wire is used in the smallest facilities (sheds, barns, garages, etc.) 120/240-V, Single-Phase, 3-wire is used in single family residential & small commercial spaces 120/208-V, Single-Phase, 3-wire is typically found within a building that takes 3-phase service 120/208-V, 3-Phase, 4-wire is a widely used 3-phase arrangement for all but very large facilities 277/480-V, 3-Phase, 4-wire is used in large buildings where lighting is primarily fluorescent or HID (multi-story office building, large single-level or multi-story industrial buildings)

What is a pro forma? A. Providing free service in the interest of a public or deserving cause B. A fixed-price set by an Owner for a construction project C. A detailed cost description that outlines each item to be included in a project D. Assumed, forecasted, or informal information presented in advance of the actual or formal information.

D A pro forma is assumed, forecasted, or informal information presented in advance of the actual or formal information. The common objective of a pro forma document is to give a fair idea of the cash outlay for an anticipated occurrence. Pro forma financial statements give an idea of how the actual statement will look if the underlying assumptions hold true. Latin for, according to form or for form's sake.

What is the best definition of occupant load? A. The estimated amount of persons assumed to occupy a space at any given time B. The maximum number of persons permitted in a building at any given time C. The average number of persons permitted in a building at any given time D. The number of persons for which the means of egress of a building or portion thereof is designed

D According to the 2012 IBC, Occupant load is the number of persons for which the means of egress of a building or portion thereof is designed.

A 4-story building is located in a highly seismic zone and the engineer recommends a structural system that allows the superstructure and substructure to move somewhat independently from each other. What is this system called? A. Lateral rolling B. Underpinning C. Tuned Mass Damping D. Base Isolation

D Base isolators allow the superstructure and substructure to move somewhat independently when lateral forces are applied.

During what era a did urban design aesthetics first come to the forefront of the public consciousness? A. Ancient Greece B. Ancient Rome C. Dark Ages D. Renaissance E. Cold War

D During the renaissance (14th-17th centuries in Europe) city planners began to focus on the aesthetics of the cities they were planning. Although city defense was still important, planners began to think of creative ways to integrate beauty and function.

A soil investigation for a building site reveals that the soil type is sandy clay and that bentonite is present. Which of the following foundation types would be most appropriate? A. Spread Footings B. Mat Foundation C. Belled Piers D. Grade Beam on Piers

D Grade beams on piers are used where expansive soil such as bentonite is present. The beams transfer the building weight to the piers, which are commonly placed on bedrock. Voids under the beams allow the soil to expand without heaving the foundation. Each soil type has a certain bearing capacity, which is the load (measured in pounds per square foot or kilopascals) from a building foundation that the soil can resist. Of the various soil types, bedrock and sedimentary rock have the highest bearing capacities.

The project site for a 10-story building contains soils that have a low bearing capacity. During the schematic phase it's assumed column footings will be used. However, the column footings have grown to a size that is now uneconomical. What type of foundation system may make sense to switch to? A. Slab on Grade B. Pile C. Caisson D. Mat

D In cases where column footings become increasingly large, it may be more economical to switch to a mat foundation (also referred to as a raft foundation)

What has the highest embodied energy? a.) Lumber b.) MDF c.) Steel d. ) Aluminum

D Lumber = 129 Btu/lbm MDF = 5130 Steel = 13,790 Aluminum = 82,320

Which of the following statements is INCORRECT? A. A 1.5% slope is suitable for rough paving B. Landscaped areas near buildings should have at least a 2% slope away from the structure. C. A safe sidewalk slopes no more than 2.5% D. Roads in northern climates can safely have up to a 12% grade.

D Most roads should be kept at a grade less than 10%; very short roads and parking garage ramps are exceptions. In northern climates, where snow and ice are a problem, it is even more important to maintain gentle slopes. A 12° o grade would not be safe and could make driving difficult.

A building that serves as an essential facility would fall under what occupancy category? A. I B. II C. III D. IV E. V

D Occupancy Categories range from I to IV I - Low hazard to human life (agricultural facilities, minor storage facilities) II - Structures not listed in categories I, III and IV (residential) III - Substrantial hazard to human life (Daycare facilities with a capcity > 150, Elementary or secondary > 250) IV - Essential facilities (Hospitals, Fire, ambulance, police stations)

Which of the following are examples of a fine aggregate? Check any that apply. A. Crushed rock B. Gravel C. Pebbles D. Sand E. Bedrock

D Sand is the only listed item that is considered a fine aggregate. Fine aggregates have particles up to a minimum size of 0.075 mm.

To allow buildings to receive light and air, zoning ordinances implement ____________ requirements. A. FAR B. Easements C. Building Height D. Setbacks

D Setbacks are the primary provision which zoning ordinances use to ensure that buildings receive light and air.

In a project you decide to use a passive cooling strategy of stack ventilation. What is important to maintain? A. A warm attic of loft space B. An inside temperature warmer than the inside temperature C. A warm basement temperature D. An outside temperature cooler than the inside temperature.

D Stack ventilation only works when the outside temperature is cooler than the inside. In the stack effect warm inside air is pushed up and out of the building by air entering from the outside low points. The cooler outdoor air pushes the hot air up into the stack.

In a project you decide to use a passive cooling strategy of stack ventilation. What is important to maintain? A. A warm attic or loft space B. An inside temperature warmer than the inside temperature C. A warm basement temperature D. An outside temperature cooler than the inside temperature

D Stack ventilation only works when the outside temperature is cooler than the inside. In the stack effect warm inside air is pushed up and out of the building by air entering from the outside low points. The cooler outdoor air pushes the hot air up into the stack.

Doubling the number of sources at a given intensity would result in a A) drop of 3 dB B) gain of 6 dB C) drop of 6 dB D) gain of 3 dB

D The answer is gain of 3 dB. Doubling the source power results in an increase of 3 dB.

The schedule of values is a statement reflecting the amounts to be allotted for the principal divisions of the work. Who prepares and submits the schedule of values? A. The Contractor submits the schedule of values to the Owner B. The Owner submits the schedule of values to the Contractor C. The Architect submits the schedule of values to the Contractor D. The Contractor submits the schedule of values to the Architect

D Where the Contract is based on a stipulated sum or Guaranteed Maximum Price, the Contractor shall submit to the Architect, before the first Application for Payment, a schedule of values allocating the entire Contract Sum to the various portions of the Work and prepared in such form and supported by such data to substantiate its accuracy as the Architect may require. This schedule, unless objected to by the Architect, shall be used as a basis for reviewing the Contractor's Applications for Payment.

What is Photochromic glazing?

Darkens under the direct action of sunlight, in the same way that some sunglasses do. As the light intensity increases, the window becomes darker. This type of glazing does not offer the control of electrochromic glazing. For example, there could be times when clear glazing is desirable, such as on a cold, sunny day.

One office machine produces 70 dB of sound, and a second machine generates 76 dB. What is the combined sound intensity level of the two office machines? A. 77 dB B. 95 dB C. 118 dB D. 146 dB

Differences between Sources 0 or 1 dB = 3 dB to add to value 2 or 3 dB = 2 dB to add to value 4 to 8 dB = 1 dB to add to value 9 or more dB = 0 to add to value *value is larger dB of the two In this case add 1 dB to 76 which equals 77 dB

When are Direct Contact Water Heaters appropriate?

Direct contact water heaters can be up to 99% efficient when the inlet water temperature is below 59°F. They also produce lower emissions of carbon monoxide and nitrous oxides than other heating systems. Because direct contact water heaters are a high-cost alternative, they are best used where there is a continuous demand for hot water, such as for food processing, laundries, and industrial purposes.

Alright, so apparently there are three types of Electrochromic Glazing. Name the three and describe it.

Electrochromic uses an inorganic ceramic thin film coating on glass and can be manufactured to range from transparent to heavily darkened (tinted). However, it is never opaque, so it cannot be used as privacy glass. It is intended for control of light, ultraviolet energy, and solar heat gain. The amount of tinting is not just an on or off condition; it can be controlled with a simple rheostat switch. Suspended particle device glazing (SPD glazing) uses a proprietary system in which light-absorbing microscopic particles are dispersed within a liquid suspension film, which is then sandwiched between two pieces of transparent conductive material. The appearance of the product can range from clear to partially darkened to totally opaque, so it can be used for privacy as well as for light control and energy conservation. It can also be controlled with a rheostat. Polymer-dispersed liquid crystal film glazing is created by placing the polymer film between two pieces of glass. The transparency can range from transparent to cloudy white. In its translucent state it offers total visual privacy but still allows a significant amount of light to pass through so it cannot be used for exterior light control. All types of electrochromic glazing are expensive, but the first two types offer the potential for significant energy savings, in the range of 20 % to 30 %.

What is a veiling reflection?

When a reflected glare interferes with a viewing task.

What is a phase change system?

Hot water systems can also take advantage of phase change materials as the collec tor fluid. As discussed in Chap. 51, phase change materials store large amounts of latent heat as well as sensible heat.

What are the four Risk (or occupancy) Categories given in the IBC during an earthquake?

I: Miscellaneous Structures II: Standard Occupancy Structures III: Hazardous Structures IV: Essential Structures Each of these risk categories is assigned an importance factor from 1.00 to 1.50 depending on how important it is for the structures in those categories to remain undamaged after a major earthquake. Hospitals, fire stations, and power generating stations, for example, are in risk category IV and have an importance factor of 1.50, while minor storage facilities, small office buildings, and retail stores may be in risk category I or II and have importance factors of 1.00.

What is a means of egress?

IBC defines means of egress as a continuous and unobstructed path of vertical and horizontal egress travel from any point in a building or structure to a public way. The means of egress consists of three parts. 1) exit access 2) exit 3) exit discharge

A room in an office contains eight machines, each of which produces 73 dB of sound. What is the over all sound level? A. 78 dB B. 82 dB C. 87 dB D. 92 dB

IL = source quantity + 10 log (n) = 73 dB + 10 log (8) =82 dB

When are handrails required on a ramp?

If the ramp has a rise greater than 6 inches or is longer than 72 inches

What are the three types of rock?

Igneous Rocks - Formed from the solidification of molten rock. Granite is a type of igneous rock. Sedimentary Rocks - consist of consolidated products of rock disintegration, sea shells, and various clays and silts. Sandstone and limestone are examples of common sedimentary rocks. Metamorphic Rocks - formed of either igneous or sedimentary rocks that have been altered by pressure or intrusion of molten rock or other liquids over a long period of time. Marble and slate are metamorphic rocks.

What is quenching?

Involves heating the metal (most often steel) to a certain temperature and then rapidly cooling it by complete submersion in water or some other liquid. This strengthens the steel.

What is ponding?

It occurs when a roof deflects enough to prevent normal water runoff. Instead, water collects in the middle of the span. With the added weight, the roof deflects a little more, which allows additional water to collect, which in turn causes the roof to deflect more. The cycle continues until structural damage or collapse occurs.

What are the differences between K-series, LH series, and DLH series for Open Web Steel Joists and Joist Girders?

K-series spans up to 60 ft LH (Long span joist) - series - come in depths of 18 - 49" and span up to 96 ft. DL-series (The deep, long span joists) come in depths of 52 in and 72 in and span up to 144 ft.

What is Low-e glass? for real..

Known as low-emissivity glass or low-e glass, is double glazing that includes a thin film or coating placed somewhere in the glazing cavity. The film or coating allows both visible and near-infrared radiation to be transmitted through the glass. However, as objects in the room are heated and emit longwave radiation, the film or coating prevents the loss of this heat; instead, the heat is reflected back into the room.

What is a Karst?

Landscape in which soluble rocks such as limestone, dolomite, and gypsum have dissolved, forming caves and other voids below the surface. These voids can cause sinkholes on the surface.

What is a closed-loop active system?

One of the most common types of solar heating systems for both residential and commercial applications. A separate, nonfreezing fluid is circulated by pumps through the solar collectors and into a heat exchanger where the domestic hot water is heated. A differential controller senses when the temperature of the collector is lower than that of the stored water and turns the pumps off. This system is flexible and provides control but suffers from some loss in efficiency because of the need for a heat exchanger.

What does the ratio 1:2:4 mean when talking about concrete mixing?

One part cement, two parts sand, and four parts gravel.

What is a sabin?

One square foot of 100% absorbing material has a value of one imperial Sabin. One square metre of 100% absorbing material has a value of one metric Sabin.

Okay what are hardwoods and softwoods?

Opposite to what you might think coniferous trees are soft woods and deciduous trees are hard woods. For finishing architectural woods, hardwoods are almost exclusively used because of their durability and their rich finish.

What is Micro-Irrigation?

Refers to the utilization of moisture sensors, rain shutoffs, drop irrigation, and a schedule to help reduce water consumption.

What is a dual-condenser cooling?

Refrigeration equipment uses two condensers instead of one. When heat is needed in the building, the heat recovery condenser is used, which sends the waste heat to fan coil units or other devices. When the heat is not needed, the heat rejection condenser sends heat to the cooling towers.

The equation Te = To + aEt / ho - 7°F represents what relationship?

Te = sol-air temperature = a value for outdoor temperature that combines the effects of temperature difference with solar radiation. THis value can be used as the ΔT value in heat gain calculations. To = outdoor dry-bulb temperature a = the surface's absorbance for solar radiation. Light colored surfaces are usually assumed to be 0.45; dark surfaces are assumed to be 0.90. Et is the total solar radiation incident on the surface, given in units of Btu/hr-ft² ho is the film coefficient of heat transfer by long-wave radiation and convection at the surface; this is usually assumed to be 3.0 Btu / hr-ft² - °F

How is tungsten halogen lamp better than typical incandescent lamps?

Technically speaking, tungsten halogen lamps are incandescent lamps as they have a filament with inert gas, but they have a halogen such as iodine or bromine which collects the metal burned off from the filament and settles it back on the filament extending the life of the bulb. Because the filament burns under high pressure/temperature the glass is made from quartz. Sometimes this bulb is referred to as quartz-halogen lamps.

What are Noise Reduction Coefficients?

The average of a material's absorption coefficients at 250 Hz, 500 Hz, 1000 Hz, and 2000 Hz, rounded to the nearest multiple of 0.05. the NRC has been superseded by the sound absorption average (SAA). The two are similar, and each provides a single number rating The SAA is the average of the absorption coefficients for the 12 one third octave bands from 200 Hz to 2500 Hz when tested in accordance with ASTM C423.

What is bisque?

The base unit of terracotta without the glaze.

What is a catchment area?

The catchment area is measured in terms of its "footprint" as projected onto a horizontal plane; the slope of the roof does not affect the size of the catchment area.

What is the difference between the cylinder test and the core cylinder test?

The cylinder test has molds of 6in in diameter and 12in in height curing along with the pour and are tested at weekly intervals. A core cylinder test drills a cylinder out of the pour and is tested in a laboratory.

What is the electrical impedance test?

The electrical impedance test uses proprietary meters to determine the moisture content of the concrete by measuring conductance and capacitance. Probes of the meter are placed on the concrete, and the percentage of moisture content in the slab is read out directly.

What is a met?

The energy produced per unit of surface area per hour by a seated person at rest. One met is 18.4 Btu/hr-ft² Given the average surface area of an adult, this means that at rest, the human body gives off from 360 Btu Btu/hr to 400 Btu/hr. This increases to around 700 Btu/hr to 800 Btu/hr for moderate activities like walking and work, and up to 2000 Btu/hr for strenuous exercise.

What should the tilt angle be for a PV panel in year-round optimization?

The latitude of the building location *10 -15° more if in the winter

What is emissivity?

The measure of an object's ability to absorb and then radiate heat. The emittance of an object is the ratio of the radiation emitted by a given object or material to that emitted by a black body at the same temperature.

What is the minimum allowable width of a stair and how far can handrailings project into the stairway?

The minimum width of any stair is 36 in, or 44 in when the occupant load exceeds 50. Handrails may project a maximum of 4% in on both sides of a stairway.

What is the polyethylene sheet test?

The polyethylene sheet test is a qualitative test conducted by sealing an 18 in X 18 in sheet of plastic to the floor to trap excessive moisture. After a minimum of 16 hours, a visual inspection is made of the floor and the sheet. The presence of visible water indicates the concrete is insufficiently dry for the application of finishes.

What is exit access?

The portion of the means of egress that leas to an exit. Exit access areas may or may not be protected depending on the specific requirements of the code, base on occupancy and construction type. Examples: rooms, spaces, aisles, intervening rooms, hallways, corridors, ramps, and doorways.

What is the difference between a boiler and a furnace?

The primary difference is the fact that a boiler uses water to heat a building while a furnace uses air.

What is Smelting?

The process of refining the ores to extract the pure metal. Once the basic metal is obtained, it usually undergoes further treatment to eliminate any impurities that might affect its use.

What is effective aperture? (EA)

The product of two variables.. 1) Visible Light Transmittance (VLT) - the percentage of visible light that passes through a glazing material. 2) Window-to-Wall Ratio (WWR) is the net glazing area in a room or space divided by the gross exterior wall area. It does not include window frames or mullions. Small, punched windows have low WWRs, while large, continuous windows have high WWRs. Generally, an EA of between 0.20 and 0.30 provides good daylighting. Thus, if the glazing has a low VLT, the size of the window should be increased. For best uniform light distribution, use continuous windows instead of punched windows.

What is the daylight factor? (DF)

The ratio of illuminance at a point on a horizontal surface indoors to the illuminance at a point on a horizontal surface outdoors and fully open to the sky, measured at the same time under overcast skies; direct sunlight is excluded. The DF is expressed as a percentage, but the percent sign is sometimes omitted (for example, a DF of 2% is sometimes expressed as a DF of 2). Various DFs are recommended for different tasks, ranging from about 1/5% for ordinary visual tasks to about 4% for difficult visual tasks such as drafting. At a DF of about 5% or more, glare and excessive heat gain and heat loss can become issues.

What is efficacy?

The ratio of luminous flux emitted to the total power input to the source and is measured in lumens per watt.

What is a viewed angle?

The solid angle formed between the measuring position and the outer edges of the object. For example, when sitting close to a fireplace, a person experiences relatively high radiant heat because the fireplace occupies a large angle of view relative to the body. When the person sits across the room, the same fireplace occupies a much smaller angle of vie~ç so it will not feel as warm.

What is Reverberation Time?

The time it takes the sound level to decrease 60 dB after the source has stopped producing the sound. It is calculated by dividing the Volume by Total Absorption T = 0.05( V / A ) = 0.05 ( V / aS )

What is warehouse matching, sequence matching, & blueprint matching?

There are also three ways panels can be assembled within a room to complete a project. Warehouse is the cheapest way of assembling veneers. Premanufactured panels, normally 4 ft wide by 8 ft or 10 ft long are assembled from a single flitch that yields from six to twelve panels. They are field cut to fit around doors, windows, and other obstructions, resulting in some loss of grain continuity. Sequence matching uses panels of uniform width manufactured for a specific job and with the veneers arranged in sequence. If some panels must be trimmed to fit around doors or other obstructions, there is a moderate loss of grain continuity. Blueprint matching is the most expensive. The panels are manufactured to precisely fit the room and line up with every obstruction so that grain continuity is not inter rupted. Veneers from the same fitch are matched over doors, cabinets, and other items covered with paneling.

What is the difference between Thermal Conductivity, k, and Conductance, C, and Resistance, R ?

Thermal Conductivity, k, is the rate at which heat passes through 1 ft² of a 1 in thickness of the material when the temperature differential is 1°F. Conductance, C, is the same property when the material is a thickness other than 1 in. Resistance, R, of a material, is the number of hours needed for 1 Btu to pas through a material of a given thickness when the temperature differential is 1°F. Total conductance and total resistance are related by.. R = 1 / C

What is the difference between the thermoset plastics and thermoplastic materials? (yeah I know)

Thermoset Plastics - are cured by being largely dependent on proper installation, subjected to heat they harden into a permanent shape. Once they have set, if they are heated they will char rather than melt again, so they cannot be remolded. On the other widespread use hand, thermoplastic materials, also called thermo plastics, can be repeatedly softened by heating and hardened by cooling, so they can be remolded. Thermoset roofing materials include EPDM and CSPE. Thermoplastic roofing materials include PVC and TPO, as well as various hybrid blends.

What is parameter method, matrix method, and unit cost method?

These are methods for calculating the cost of construction. parameter method is typically used during the design development phase and early stages of construction document production. It involves an expanded itemization of construction quantities and assignment of unit costs to these quantities. For example, instead of using one conglomerated sum for floor finishes, the cost is broken down into carpeting, vinyl tile, wood strip flooring, unfinished concrete, and so forth. Using an estimated cost per square foot, the cost of each type of flooring can be estimated based on the area. Matrix Costing is drawn showing the various alternatives along one side and the individual elements that combine to produce the total cost of the alternatives on the other side. For example, in evaluating alternatives for workstations, all the factors that lead to the final cost could be compared. These factors might include the cost of custom-built versus pre-manufactured workstations, task lighting that could be planned with custom built units versus higher wattage ambient lighting, and so on. Unit cost method is broken down into its individual building components and the labor needed to install them. Contractors typically use this method of estimating when they are determining a bid or negotiated price for the project.

What is an Synclastic Shell or anticlastic shell (hyperbolic paraboloid)?

Thin-shell domes are very rigid and efficient structures. They are stable for either symmetric or asymmetric loads. In theory, they behave like the frame domes discussed in a previous section, but because they consist of one continuous surface, each infinitesimal portion is resisting compression, tension, and shear See Fig. 18.17. Compression is acting in the lines of the meridian, and either tension or compression is acting in the hoop direction. Shear is therefore developed in any section to keep the structure in equilibrium.

What is Thermochromic?

This glazing changes darkness in response to temperature; it becomes translucent when it reaches a certain temperature. Like photochromic glazing, the change is an inherent property of the material, so this technique offers less control than electrochromic glazing does.

What is the Elevator Operation : Group Automatic Operation?

This is simply the control of all elevators with programmable microprocessors to respond to calls in the most efficient manner possible, taking into account all the variables involved. In addition, such things as the time of day or day of the week can be included in the programming. This provides precise response to any building's needs.

What is the Elevator Operation : Destination Floor Guidance System?

This kind of system uses computer control with artificial intelligence to put riders into the same car who are going to the same floor or to floors near each other. With this system, each rider selects a destination floor in the lobby, either on a wall panel or on a kiosk, and the control system assigns a particular car based on what other riders have selected. Once in the car, the riders do not press any destination buttons; the car stops at the floors that were chosen in the lobby. The system reduces total travel time to the destination floor and results in fewer stops for each car.

If plywood has a structural grading of 32/24 then what does this mean?

This means that the plywood can span 32 inches between roof supports and 24 inches between floor supports.

What is Spectrally Selective Glazing?

Transmits a high proportion of the visible solar spectrum while blocking up to 80% of the heat from the infrared portion of the spectrum. Used with a low-e coating, a double-glazed window can achieve an SHGC of approximately 0.25.

What is a Flat-Plate Heat Recovery Unit?

Two ducts of various designs - one for incoming air and one for exhaust air—separated by a thin wall that facilitates the heat transfer These units can only exchange sensible heat, and they offer no humidity control.

What are the characteristics of the five types of Portland Cement?

Type I - General purpose. Fairly high C3S content for good early strength development. General construction (most buildings, bridges, pavements, precast units, etc). Type II - Moderate sulfate resistance. Structures exposed to soil or water containing sulfate ions. Type III - High early strength. Rapid construction, cold weather concreting. Type IV - Low heat of hydration (slow reacting). Massive structures such as dams. Now rare. Type V - High sulfate resistance. Structures exposed to high levels of sulfate ions.

What are the Construction Types?

Type I - Non-Combustible Materials and Structure Type II - Non-Combustible Exterior and Components Type III - Non-Combustible Exterior Type IV - Heavy Timber Construction Type V - Combustible Construction

Equation U = 1 / ΣR represents what relationship?

U stands for "Overall Coefficient of Heat Transmission" - it is a coefficient used to calculate total heat loss. The summation of resistances is from the different materials used in an assembly.

What is Ultra High Performance Concrete (UNPC)?

Ultra high performance concrete (UNPC) is a type of concrete characterized by high strength, low water absorption, and high resistance to waterborne and airborne chemical degradation. Compressive strength can range from 17,000 psi to 25,000 psi with flexural strength from 3600 psi to 6000 psi. UHPC is comprised of cement, sand, water, silica fume, and plasticizers along with a small percentage of alkali resistant glass fibers or steel fibers. The unique properties of UHPC are achieved by using extremely small particle sizes and unique particle chemistry mixed, vibrated, and cured in the factory under controlled conditions. Because of the unique characteristics of the material it is typically delivered precast and not mixed on the job site.

What are Variable Refrigerant Flow systems? (VRF)

Uses a single compressor and condenser unit located outdoors, connected to multiple evaporators located in different zones of the building. Refrigerant is supplied to each zone instead of the chilled and heated water used in more traditional all water systems. Each evaporator is individually controlled, and the amount of refrigerant supplied to each zone varies based on the needs of the zone.

What is a Thermal Energy Storage System?

Uses water, ice, or rock beds to store excess heat or coolness for use at a later time. Thermal storage makes it possible to manage a building's energy needs over cli matic temperature swings throughout the day or week, and it allows the use of less expensive, off-peak energy costs to cool. For example, in the summer, chillers can cool water at night, when utility rates are typically lower and the cooling needs of the building are not as great as they are during the day. During the next day, the stored coolness can be used to minimize the energy needed for cooling.

What is running match, balance match, and center match?

Veneers must be glued to rigid panels to make installation possible. The method of doing this is the next consideration in specifying paneling If the veneers are bookmatched, there are three ways of matching veneers within a panel. A running match simply alternates bookmatched veneer pieces regardless of their width or how many must be used to complete a panel. Any portion left over from the last leaf of one panel is used as the starting piece for the next. In a balance match, veneer pieces are trimmed to equal widths; there may be an odd or even number of veneer pieces in each panel. A center match has an even number of veneer leaves of uniform width so that there is symmetry about a veneer joint in the center of the panel.

Okay wtf is the difference between welding, brazing, and soldering?

Welding is joining two metals together but heating them past their melting point. Brarcing is the joining of two metals at an intermediate temperature using a nonferrous filler metal with a melting point that is above 800°F but lower than welding. Brazing is usually used for brass, bronze, and some aluminums. It results in a clean joint, although some buffing may be required if a completely smooth joint is desired. Soldering is the joining of two metals using lead-based or tin based alloy solder filler metal that melts below 500°F.

What are additive colors and subtractive colors?

When colors created with light are mixed, the wavelengths of both colors are present in the resulting light. For this reason, colors created with light are called additive colors. The three primary colors of light are red, green, and blue. The color of a physical object, on the other hand, is conveyed by the wavelengths of light that the object absorbs, or subtracts, from the light that strikes it. When white light strikes a blue object, the object absorbs the wavelengths of light except that of blue light, which is reflected to the eye. For this reason, colors created with pigments are called subtractive color. The primary colors of pigment are blue, yellow, and red.

What produces white light in terms of the visible spectrum?

White light is produced when a source emits approximately equal quantities of energy over the entire visible spectrum.

How far does daylight penetrate relative to the distance from floor to top of window? How much is it improved by a light shelf?

With a standard windows that has no overhang protection or light shelf, the effective daylighted zone extends about 1.5 times the window head height into the room. When a light shelf is used, the effective daylighted zone is from 2.0 to 2.5 times the window head height.

What is the Elevator Operation : Selective Collective Operation?

With this system, the elevator remembers and answers all calls in one direction and then reverses and answers all calls in the opposite direction. When the trip is complete, the elevator can be programmed to return to a home landing, usually the lobby.

What is the difference between Cast Iron and Wrought Iron?

Wrought iron is iron with a very low carbon content Qess than about 0.30%) and a substantial amount of slag. It is similar in chemical composition to low-carbon steel, but most of the impurities are in the slag, which is mechanically mixed with the iron. Because of its low carbon content, wrought iron is soft, duc tile, and resistant to corrosion. Its use in construction is limited to ornamental iron work such as gates, grilles, and fences. Cast iron is iron with a carbon content above 2%. With this high percentage of carbon, it is very hard, but brittle. Cast iron was used extensively in the nineteenth century for columns and beams in such structures as the Crystal Palace, mill buildlings in New England, and some of the early commercial build ings in New York City

What is Silicon-Controlled Rectifier (SCR)?

control system (also called a t4yristor control system) provides variable DC voltage to a DC traction motor. This system provides good drive and leveling control, but it also has a low power factor and high thermal losses. The older AC system had many disadvantages, and the DC system is generally used.

What is Autoclaved Aerated Concrete (AAC)?

is a precast concrete product manufactured by adding aluminum powder to concrete, hardening it in molds, and then curing the molds in a pressurized steam chamber (auto clave). The resultant blocks have approximately one-fifth the density of conventional concrete. They are typically manufactured in blocks 10 in high by 25 in long and in thicknesses of 4 in, 8 in, and 10 in. The blocks are laid with a thinset mortar and can be cut and shaped with woodworking tools. Unreinforced and reinforced panels of ACC are also produced for use as floor, roof and wall panels.

What is a rigid frame?

is a structural system in which the vertical and horizontal members and joints resist loads primarily by flexure, and in which moments are transferred from beams to columns.

What is a deadman?

is a timber, plate, or similar object placed perpendicular to the face of the retaining wall and that serves to anchor the wall by means of earth friction or pressure.

What is the Angle of Natural Repose?

is the maximum practical angle for changing grades without using retaining walls or other stabilization techniques. However, even though a slope may conform to the angle of repose of a material, it may still be unsuitable to prevent erosion or allow for the desired type of landscaping.

What are the ingredients of Portland Cement?

lime, silica, iron oxide, and alumina (also called aluminum oxide) under strictly controlled conditions. Portland cement interacts chemically with water to form a paste that binds the other aggregate particles together in a solid mass. Cement is supplied in bulk or in 94 Ibm bags containing one cubic foot of cement mix.

What does 6 X 6 -- W1.4 X 1.4 mean?

means that the grid is 6 in by 6 in, and the size of the wire is 1.4 hundredths of a square inch, or 0.014 in2. The letter preceding the gage is either W for smooth wire or D for deformed wire. See Carbon Fiber Concrete section for information on carbon fiber mesh, which can also be used for reinforcement of slabs.

What is the slump test?

measures the consistency of the concrete. In this test, concrete is placed in a 12 in high truncated cone, 8 in at the base and 4 in at the top. It is compacted in the cone by hand with a rod, and then the mold is removed from the concrete and placed next to it. The distance the concrete slumps from the original 12 in height is then measured in inches. The amount of slump desired depends on how the concrete is going to be used, but it is typically specified to be in the range of 2 in to 6 in. Too much slump indicates excessive water in the mix, and a very small slump indicates that the mixture will be difficult to place properly.

What is Alkali-Silica Reaction (ASR)?

more commonly known as "concrete cancer", is a swelling reaction that occurs over time in concrete between the highly alkaline cement paste and the reactive non-crystalline (amorphous) silica found in many common aggregates, given sufficient moisture. The expansion of concrete through reaction between cement and aggregates was first studied by Thomas E. Stanton in California during the 1930s with his founding publication in 1940.[3] This reaction causes the expansion of the altered aggregate by the formation of a soluble and viscous gel of sodium silicate (Na2SiO3 · n H2O, also noted Na2H2SiO4 · n H2O, or N-S-H (sodium silicate hydrate), depending the adopted convention). This hygroscopic gel swells and increases in volume when absorbing water: it exerts an expansive pressure inside the siliceous aggregate, causing spalling and loss of strength of the concrete, finally leading to its failure. ASR can cause serious cracking in concrete, resulting in critical structural problems that can even force the demolition of a particular structure.

At what size is a wire considered to be a cable?

no. 6 AWG or larger, or several conductors assembled into a single unit, is referred to as a cable.

What is the inverse square law?

states that the intensity of a force or energy (such as a sound wave) at a given point is inversely proportional to the square of the distance from the source of that energy. It is derived from Eq. 19.2. I1 / I2 = r2 ^ 2 / r1 ^ 2

What is the minimum water-cementitious ratio for concrete?

the minimum water-cementitious materials (w/cm) ratio is about 0.35 to 0.40 by weight. Based on the weight of water, this works out to about 4 gal to 4.5 gal of water per 94 Ibm sack of cement.

What is the K-slump test?

uses a 3/4 in tube that contains a floating scale. The tube is placed on the wet concrete, and the scale is pushed into the mixture and released. The distance the scale floats out directly measures the consistency of the concrete, comparable to the slump measure by the traditional slump test.

What are the dates of the winter solstice, summer solstice, spring equinox and fall equinox?

winter solstice = December 21 summer solstice = June 21 spring equinox = March 21 fall equinox = September 21


Set pelajaran terkait

The Natural Anticoagulants and the Fibrinolytic System

View Set

Chapter 14: Care of the Patient with a Neurologic Disorder

View Set

COMBO "GRADE 8 - HISTORY - THE CRUSADES & CAUSES OF CRUSADES

View Set

Nursing Process- Nursing Diagnosis

View Set

3.1 The First Industrial Revolution

View Set

Marketing Management - Chapter 13

View Set